You are on page 1of 93

DEPARTMENT OF EDUCATION

REGION III
Division of City of San Fernando
PAMPANGA HIGH SCHOOL
PHS Blvd., Brgy. Lourdes, City of San Fernando, Pampanga

in

Grade 11
Prepared by:
Ms. Cherry Cordero
Ms. Jerica Nicole Flores
Mr. Jay Vincent Quizon
Mr. Peter Paul Carreon
Mr. Jessie Manguera
Mr. Eugene Borbajo
Table of Contents

Week Contents Page

1 Concept of Functions 1

2&3 Rational Functions 16

4 Inverse Functions 39

5 Exponential Functions 52

6 Graphing Exponential Functions 69

7 Logarithmic Functions 81
SELF-INSTRUCTIONAL PACKETS
GENERAL MATHEMATICS GRADE 11

School Teaching Dates/ Week


August 24 – August 28, 2020
PAMPANGA HIGH SCHOOL (Week 1)
Teacher GENERAL MATHEMATICS TEACHERS Quarter First

I. OBJECTIVES

A. Content Standards
The learner demonstrates understanding of key concepts of functions.

B. Performance Standards
The learner is able to accurately construct mathematical models to represent real-life
situations using functions.

C. Learning Competencies
The learner
1. represents real-life situations using functions, including piece-wise functions;
(M11GM-Ia-1)
2. evaluates a function; (M11GM-Ia-2)
3. performs addition, subtraction, multiplication, division, and composition of
functions; (M11GM-Ia-3) and
4. solves problems involving functions. (M11GM-Ia-4)

D. Objectives
At the end of the lesson, the learners should be able to:
1. represent real-life situations using functions, including piece-wise functions;
2. evaluate a function;
3. perform different operations and composition of functions; and
4. show appreciation by solving problems involving functions.

II. CONTENT
CONCEPT OF FUNCTIONS

Learning Resources

A. Reference
Learner‟s Material in General Mathematics, pp. 1-23
Teacher‟s Guide in General Mathematics, pp. 1-22
B. Other Learning Resources
Our Lady of Fatima University, General Mathematics Online Book, pp. 2-10

III. PROCEDURES

WEEK 1: Day 1

A. Reviewing previous lesson or presenting the new lesson

Welcome to the world of General Mathematics! Are you familiar with functions?

Many people are not aware that function is described in certain phenomena and situations.
Function is one of the core concepts in Mathematics as well as in our daily lives. Just for instances,
functions in Mathematics can:
- compute the height of a plant,
- its growth every period, and
- even figure an employee‟s salary savings in a month.

Page 1
In your junior high school mathematics, you have learned about linear, quadratic and
polynomial functions.

In this lesson, you will learn more applications of functions in real-life situations. Let us start by
defining a function.

REMEMBER ME?

Definition of a function:
1. It is a relation where each element in the domain (x) is related to only one value in the
range (y) by some rule.
2. No two ordered pairs have the same domain (x-value) but different range (y-values).
3. Usually represented by the equation y = f(x).

Functions can be represented or modeled thru table of values, mapping diagram, set of
ordered pairs and graph.

DIRECTIONS: Put a check mark on the blank provided before each number if it is a function.
Copy ALL and write the answers on your Gen Math notebook.
__________ 1. f = {(0, -1), (2, -5), (4, -9), (6, -13)}
__________ 2. r = {(a, 0), (b, -1), (c, 0), (d, -1)}
__________ 3. g = {(5, -10), (25, -50), (50, -100)}
__________ 4. t = {(-2, 0), (-1, 1), (0, 1), (-2, 2)}
__________ 5. h = {(1,3), (2,6), (3,9), (4,12),… }

DIRECTIONS: Determine if the given mapping diagram represents a function. Write on the
blank provided before each number if it is a function „F‟ and „NF‟ if it is not. Copy ALL and
write the answers on your Gen Math notebook.

___6. X Y ___7. X Y
f g
1 3 5
2 5 6 0
3 9 7
4 17 8 1
5 33 9

___8. X Y
h Score Description
7 11
8 Naol perfect!
13
5-7 Bongga ka!
2 17
19 3-4 Pwede na besh
1 23 Practice pa more
0-2
on page 12 sec J

Now, check your work by turning to page 13 for the key to correction. How many correct
answers did you get? Rate your result using the table above. If your score is at least 4 out of 8, you
may now proceed to next part of the discussion.

In this module, you will learn to evaluate, to perform operations, and solve problems involving
Function. Let‟s get ready!

Page 2
B. Establishing a purpose for the lesson

Peter Gustav Lejeune Dirichlet


(February 13, 1805 – May 5, 1859)
A German mathematician credited for the modern
definition of Functions…
"to any x there corresponds a single finite y"
REFLECT: Who is/are your X‟s in life? Y?

C. Presenting examples/instances of the new lesson


Do you miss the outside world already? Are you now ready to meet “functions” in the real
world? Despite the COVID-19 pandemic, we need to learn how to function well while learning
at home.

Lesson 1: Functions as Representations of Real-life Situations.

Some application of Function represents situations such as sales and expenditures of


certain products, the number of hours spent in studying and the grades earned by the
students.

Check out the following examples.

Problem 1. Give a function C that can represent the cost of buying x meals, if one meal costs
₱40.

Function: C(x) = 40x

TAKE NOTE: Since each meal costs ₱40, then the cost function is C(x) = 40x.
This is a very easy problem, right?

Problem 2. Betty and Kris are best of friends. They decided to


have a part time income. They buy beads and other materials in
Divisoria and make bracelets to sell to their other friends. Their
starting capital is worth ₱1000 which can make 100 pieces of
bracelets. If they sell it for ₱20 each, how much is their profit?

Function: P (n) = 20n – 1000


Final Answer: ₱1000

TAKE NOTE: To represent the amount of profit (P), we used n for the number of pieces of
bracelets made and a constant of Php1000 for the capital. The equation for this situation is
represented by P (n) = 20n – 1000.Therefore, the profit is;
P (n) = 20n – 1000 Represent the function.
= 20 (100) – 1000 Substitute n bracelets
= 2000 – 1000 Simplify
P (n) = 1000 So their profit is ₱1000.

Can you represent a function in real-life? Try to make your own function that can be
applied in real-life situation.

In the next part of the lesson, you will be learning another type of function.

Page 3
Some situations can only be described by more than one formula, depending on the value
of the independent variable.

A piecewise function is a function defined by multiple sub-functions, where each sub-


function applies to a certain interval of the main function‟s domain. It behaves differently
across a sequence of intervals.

Problem 3: A user is charged ₱300 monthly for a particular


mobile plan, which includes 100 free text messages.
Messages in excess of 100 are charged ₱1 each. Represent
the amount a consumer pays each month as a function of the
number of messages m sent in a month.

Piecewise Function: ( ) {
TAKE NOTE: 1. The input value is the no. of text messages and the output is amount of
monthly bill.
2. Let t(m) represent the amount paid by the consumer each month.
3. Mark the function with t(m) = {
4. Put the conditions on the right side starting with the word “if” using inequality
symbols. Between 0 to 100 messages is represented by while
more than 100 messages by
5. Represent the first function with a fixed amount of “300” for messages
between 0 to 100 while the second function with “300 + m” for messages
more than 100.

Problem 4. A jeepney ride costs ₱8.00 for the first 4


kilometers, and each additional integer kilometer adds
₱1.50 to the fare. Use a piecewise function to represent
the jeepney fare in terms of the distance d in kilometers.

Piecewise Function: ( ) {
, -

TAKE NOTE: 1. The input value is distance and the output is the cost of the jeepney fare.
2. Let F (d) represents the fare as a function of distance
3. Mark the function with F(d) = {
4. Put the conditions on the right side starting with the word “if” using inequality
symbols. Between 0 to 4km is represented by while more than 4km
by
5. Represent the first function with a fixed value “8” for the fare between 0 to
4km while the second function with “8 + 1.5 [d]” for the fare more than 4km.
6. The symbol [d] denotes the greatest integer or floor function applied to d.
The floor function gives the largest integer less than or equal to d, e.g. [4.1] =
[4.9] = 4.

Can you make your own piece-wise function? Do you find this concept easy?

Images retrieved from:


 https://www.google.com/search?q=bracelet+and+beads+cartoon&tbm=isch&ved=2ahUKEwjWvt6-y-XpAhUOm0sFHZeQBDYQ2-
cCegQIABAA&oq=bracelet+and+beads+cartoon&gs_lcp=CgNpbWcQAzoICAAQCBAHEB46BggAEAgQHlC9nAFY1KwBYM6uAWgAcAB4AIABvQKIAdcPkgEHMC42L jMuMZ
gBAKABAaoBC2d3cy13aXotaW1n&sclient=img&ei=7I_XXtaYE462rtoPl6GSsAM&bih=654&biw=1366&client=firefox-b-d#imgrc=uIv5tsrwcT_78M
 https://www.google.com/search?q=jeepney+cartoon&tbm=isch&ved=2ahUKEwjYkMK4xeXpAhXYaCsKHdFrBMwQ2-
cCegQIABAA&oq=jeepney+cartoon&gs_lcp=CgNpbWcQAzICCAAyBAgAEB4yBAgAEB4yBAgAEB4yBggAEAUQHjIGCAAQBRAeMgYIABAFEB4yBggAEAUQHjIGCAAQBRA
eMgYIABAFEB46BAgAEENQ1gdYiRZg_xhoAHAAeACAAdgDiAGhE5IBBzItNC4zLjGYAQCgAQGqAQtnd3Mtd2l6LWltZw&sclient=img&ei=lInXXtj2J9jRrQHR15 HgDA&bih=65
4&biw=1366&client=firefox-b-d#imgrc=55j1qERe2YSO6M
 https://www.google.com/search?q=mobile+phones+text&tbm=isch&ved=2ahUKEwiy77SjzuXpAhXvnUsFHRtVAkUQ2-
cCegQIABAA&oq=mobile+phones+text&gs_lcp=CgNpbWcQAzIECAAQEzIICAAQCBAeEBNQugpYthZg0hpoAHAAeACAAf0CiAH-
CZIBBzAuMS4zLjGYAQCgAQGqAQtnd3Mtd2l6LWltZw&sclient=img&ei=2JLXXrKjE--7rtoPm6qJqAQ&bih=654&biw=1366&client=firefox-b-d#imgrc=Ph-AluY3KUxw8M

Page 4
DIRECTIONS: Writing functions. Solve the following scenarios. Copy ALL and write
the answers on your Gen Math notebook.
 A tree grows 20cm every year, so the height of the tree is related to its age.
1. Write the function.
2–3. Find the height of the tree if its age is 5 Score Description
years. 6 Naol perfect!
4-5 Bongga ka!
 Jason goes to Boom-na-Boom theme park where
2-3 Pwede na besh
he pays ₱10.00 admission and ₱5.00 per ride.
4. Write a function that describes this situation Practice pa more
0-1
where f(x) is the total amount of expenses. on page 12 sec J
5–6. Find the amount Jason spent after riding 12
rides.

Now, check your work by turning to page 13 for the key to correction. How many correct
answers did you get? Rate your result using the table above. If your score is at least 3 out of 6,
you may now proceed to next part of the discussion.

WEEK 1: Day 2

D. Discussing new concepts and practicing new skills #1

Functions are described by using formulas or function notations. Function of each x-


coordinate is match with only one y-coordinate. The domain stands as the set of inputs and
range as the sets of outputs is a function of f. This is a function notation ( ) read as “f of x”.
Lesson 2: Evaluating Functions

To find for a given value of is to evaluate the function f by substituting the value of in
the equation.

Illustrative Example 1: Given ( ) .


If x in the function ( ) have values equal to -1, 0, 1, and -2.
Then;

f (–1) = 2 (–1) + 3 Substitute –1 for x


= –2 + 3 Simplify
f (–1) = 1 Final Answer

f (0) = 2 (0) + 3 Substitute 0 for x


=0+3 Simplify
f (0) = 3 Final Answer

f (1) = 2 (1) + 3 Substitute 1 for x


=2+3 Simplify
f (1) = 5 Final Answer

f (–2) = 2 (–2) + 3 Substitute –2 for x


= –4 + 3 Simplify
f (–2) = –1 Final Answer

Page 5
Evaluating functions is just easy right? All you have to do is to substitute each value to
the given function.
Let us have more examples.

Illustrative Example 2: If ( ) , evaluate each;


f(5), f(-3), f(-x) and f(x+2).

f(5) = 3(5) – 1 = 14

f(–3) = 3(–3) + 1 = –8

f(–x) = 3(–x) – 1 = –3x – 1

f(x+2) = 3(x+2) – 1 = 3x + 6 – 1 = 3x + 5

Now it‟s your time to do your task! Try to answer the next challenge. Good luck!

DIRECTIONS: Evaluate the following. Copy ALL and write the answers on your Gen
Math notebook.

Given: If ( ) , find:
1-2. ( )
Score Description
3-4. ( )
12 Naol perfect!
5-6. ( )
9-11 Bongga ka!
7-8. . / 5-8 Pwede na besh
Practice pa more
9-10. . / 0-4
on page 12 sec J
11-12. ( )

Now, check your work by turning to page 13 for the key to correction. How many correct
answers did you get? Rate your result using the table above. If your score is at least 4 out of 12,
you may now proceed to next part of the discussion.

WEEK 1: Day 3

E. Discussing new concepts and practicing new skills #2

Lesson 3: Fundamental Operations on Functions

Functions are like real numbers which can be performed by addition, subtraction,
multiplication and division. Suppose f and g are functions and x is an element common to the
domains of f and g. The operations on functions are defined as follows:

 The sum of f and g, denoted by


( )( ) ( ) ( )

 The difference of f and g, denoted by


( )( ) ( ) ( )

 The product of f and g, denoted by


( )( ) ( ) ( )

 The quotient of f and g, denoted by f


( )
. /( ) ( )
( )

Page 6
Illustrative Example 1: Given ( ) , and ( ) , find:
1. ( )( ) 3. ( )( )
2. ( )( ) 4. . /( )

Solution:
1. ( )( ) ( ) ( ) Model using sum of functions
( ) ( ) Replace the value of each function
( )( ) Simplify by combining like terms

2. ( )( ) ( ) ( ) Model using difference of functions


( ) ( ) Replace the value of each function
( )( ) Simplify by changing the signs of the subtrahend

3. ( )( ) ( ) ( ) Model using product of functions


( )( ) Replace the value of each function
( )( ) Simplify using FOIL Method

( )
4. . / ( ) Model using quotient of functions
( )
( )
Replace the value of each function
( )
( ) Simplify using factoring
( )
. /( ) Cancel common factors

Illustrative Example 2: Given ( ) , and ( ) , find:


1. ( )( ) 3. ( )( ) s
2. ( )( ) 4. . / ( )
Solution:
Model using sum of functions
1. ( )( ) ( ) ( ) Replace the value of each function
( ) ( ) Simplify by combining like terms
( )( )

( )( ) ( ) ( ) Evaluate the function by replacing -2 for x


Simplify by performing the indicated operations
( )( ) Final answer

2. ( )( ) ( ) ( ) Model using product of functions


( )( ) Replace the value of each function
( )( ) Simplify using FOIL Method

( )( ) ( ) ( ) ( ) Evaluate the function by replacing -2 for x


Simplify by performing the indicated operations
( )( ) Final answer

3. ( )( ) ( ) ( ) Model using difference of functions


( ) ( ) Replace the value of each function
( )( ) Simplify by changing the signs of the subtrahend

( )( ) ( ) ( ) Evaluate the function by replacing -3 for x


Simplify by performing the indicated operations
( )( ) Final answer

Page 7
( )
4. . /( ) ( )
Model using quotient of functions
( )
( )
Replace the value of each function
( )( )
( ) Simplify using factoring
( )( ) Cancel common factors

. /( ) Evaluate the function by replacing -1 for x

( )( ) Final answer

Do you find this part of the lesson easy? Simply recall basic operations on integers in
your Junior High School Mathematics. For sure, it will be easy to solve operations on functions.
For the next part of the discussion, we will talk about composition of functions. Are you ready?

Lesson 4: Composition of Functions

The composition of the function f with g is denoted and defined by


( )( ) ( ( ))
read as “f circle g of x is equal to f of g of x”.
.
Illustrative Example:
Let ( ) and ( ) find, ( )( ) and ( )( )

Solution:

( )( ) ( ( )) Rewrite the notation as “f of g of x”.


( ) Substitute x2 + 5 for g(x).
( ) Using f(x) = 3x + 7, substitute “x2 + 5” for x.
Simplify the expression using distributive property.
( )( ) Simplify by combining similar terms.

( )( ) ( ( )) Rewrite the notation as “g of f of x”.


( ) Substitute 3x + 7 for f(x).
( ) Using g(x) = x2 + 5, substitute “3x + 7” for x.
Simplify the expression using special products.
( )( ) Simplify by combining similar terms.

DIRECTIONS: Solve the indicated operation. Copy ALL and write the answers on your
Gen Math notebook.

Given: If ( ) and ( ) , find:


Score Description
1-2. ( )( )
3-4. ( )( ) 12 Naol perfect!
5-6. ( )( ) 7-11 Bongga ka!
7-8. ( )( ) 4-6 Pwede na besh
9-10. ( )( ) Practice pa more
11-12. ( )( ) 0-3
on page 12 sec J

Now, check your work by turning to page 14 for the key to correction. How many correct
answers did you get? Rate your result using the table above. If your score is at least 6 out of 12,
you may now proceed to next part of the discussion.

Page 8
WEEK 1: Day 4

Lesson 5: Solving Problems

Analyze the given problems and perform the appropriate operations on functions;

Problem 1: Find the polynomial that represents the length


of rectangular field if the area is
(𝒙 𝟕) 𝟐𝒙𝟑 𝟏𝟏𝒙𝟐 𝟐𝟐𝒙 𝟕
square meters and the width is meters.

Solution:
𝒍 ?
Area of a rectangle
Derive the formula in terms of the length
( )
( )
Replace the function of the area and width
( )( )
( )
Simplify by factoring
meters Cancel common factors

Problem 2: A manufacturing company produced baby dresses that cost


₱75 each. If the dress is sold in x pesos and the number of dresses sold
per day is ( ), what function represents the daily profit in pesos of
the manufacturing company? If the dress is sold at ₱150, how much is
the profit per day?

Solution:
( ) ( ) Represent the profit function
Simplify using distributive property
Simplified function
( ) ( ) Substitute 150 for x
Perform the indicated operation
Final Answer

Image retrieved from:


https://www.google.com/search?q=baby+dress+cartoon&client=firefox-
bd&sxsrf=ALeKk01TK7Hvueny_CEqdeza8cKpaUgUoQ:1591322870561&source=lnms&tbm=isch&sa=X&ved=2ahUKEwjgyLDTy-
npAhUSYysKHQKZAuUQ_AUoAXoECAoQAw&biw=1366&bih=654#imgrc=tJpOqv4aChMPOM

F. Developing mastery
GENERAL DIRECTIONS: Perform the indicated operations. Copy ALL and write the answers
on your Gen Math notebook.

Given: ( ) and ( ) , find;


1−2. ( )( )
3−4. ( )( ) Score Description
5−6. ( )( ) 20 Naol perfect!
7−8. ( )( )
9−10. ( )( ) 11-19 Bongga ka!
11−12. ( )( ) 6-10 Pwede na besh
0-5 Practice pa more
Given: ( ) and ( ) , find;
13−14. ( )( )
15−16. ( )( )
17−18. ( )( )
19−20. ( )( )

Now, check your work by turning to page 15 for the key to correction. How many correct
answers did you get?

Page 9
G. Finding practical applications of concepts and skills in daily living
A function is merely a “machine” that generates some output in correlation to a given
input. So, if ( ) then, ( )
Understanding this behavior is essential to recognizing the variety of input\output
correlations in the real world. Again, all a function does is provide a mathematical way to
model or represent a situation where a certain input will give a certain output.
Here are a few examples:
 Circumference of a Circle - A circle‟s circumference is a function of its diameter.
 A Shadow - The length of person‟s shadow along the floor is a function of their height.
 Driving a Car - When driving a car, your location is a function of time. You see in
Quantum Physics, you cannot be in two places at once. Therefore, the vehicle‟s position
is a function of time.
 Temperature - Based on a variety of inputs, or factors, we get a certain temperature. So,
temperature is a function of various variables.
 Money - The amount of money you have is a function of the time spent earning it.
H. Making generalizations and abstractions about the lesson
What is the difference between a relation and function? Do you still remember this in Junior
High School Mathematics?
RELATIONS FUNCTIONS
A relation is a rule that relates values from a A function is a relation where each element in
set of values called the domain to the second the domain is related to only one value in the
set of values called the range. range by some rule.
A function is a set of ordered pairs (x, y) such
A relation is also a set of ordered pairs (x, y). that no two ordered pairs have the same x-
value but different y-values.

WEEK 1: Day 5
I. Evaluating learning

GENERAL DIRECTIONS: This part is recorded and graded. Copy ALL the given using this
format on a separate yellow sheet of paper. The yellow paper will be submitted to your Math
teachers on the day of the scheduled retrieval.

ACTIVITY 1.1 IN GENERAL MATHEMATICS


(WEEK 1)
Name: _________________Section: _________ Parent‟s Signature: ___________________
EVALUATING FUNCTION
DIRECTIONS: Evaluate the following functions. Show your complete solutions.
1. Given: ( )
a. ( ) d. ( )
b. ( ) e. ( )
c. . /
2. Given: ( )
a. ( )
b. ( )
c. ( )

ACTIVITY 1.2 IN GENERAL MATHEMATICS


(WEEK 1)
Name: _________________Section: _________ Parent‟s Signature: ___________________
EVALUATING FUNCTION
DIRECTIONS: Evaluate the following functions. Show your complete solutions . Express your
answer in simplest form.
1. Given the function ( ) , find;
a. ( ) b. ( )
2. Given the function ( ) , find;
a. ( ) b. ( )
3. Given the function ( ) , find;
a. ( ) b. . /

Page 10
4. Given the function ( ) , find;
a. ( ) b. ( )
5. Given the function ( ) , find;
a. ( ) b. ( )
6. Given the function ( ) , find;
a. ( ) c. ( ) e. ( )
b. . / d. ( )

ACTIVITY 1.3 IN GENERAL MATHEMATICS


(WEEK 1)
Name: _________________Section: _________ Parent‟s Signature: ___________________
OPERATIONS ON FUNCTION
DIRECTIONS: Perform the indicated operation. Show your complete solution .
1. ( ) – and ( )
a. ( )( ) c. ( )( )
b. ( )( ) d. . / ( )
2. ( ) and ( )
a. ( )( ) c. ( )( )
b. ( )( ) d. . / ( )
3. ( ) and ( )
a. ( )( ) b. ( )( )
4. ( ) and ( )
a. ( )( ) b. ( )( )
5. ( ) ( )
a. ( )( ) c. . / ( )
b. ( )( )

ACTIVITY 1.4 IN GENERAL MATHEMATICS


(WEEK 1)
Name: _________________Section: _________ Parent‟s Signature: ___________________
COMPOSITIONS OF FUNCTIONS
DIRECTIONS: Given the following functions, solve for the composition of each given pair of
functions. Express all answers in simplest form.
( ) ( )
( ) ( )
( ) ( )
1. ( )( ) 6. ( )( )
2. ( )( ) 7. ( )( )
3. ( )( ) 8. ( )( )
4. ( )( ) 9. ( )( )
5. ( )( ) 10. ( )( )

ACTIVITY 1.5 IN GENERAL MATHEMATICS


(WEEK 1)
Name: _________________Section: _________ Parent‟s Signature: ___________________
PROBLEM SOLVING
DIRECTIONS: Solve the following problems. Show your complete solution.
1. The volume of rectangular bar is 640 cubic units. The width is 3 units more than the height
and the length is 1 unit more than three times the height. Find the dimensions of the bar.
2. If the rectangle is square centimeters. Write a polynomial that represents
its width if its length is cm.
3. Arman traveled a distance of kilometers at a rate of kph. Determine a
function that represents the number of hours he traveled.

Page 11
J. Additional activities for application or remediation
GENERAL DIRECTIONS: Copy ALL and write the answers on your Gen Math notebook.

Remedial Activity 1.1: FUNCTION OR NOT


DIRECTIONS: Write “F” if the set of ordered pairs models a function, otherwise write “NF”.
______________1. *( ) ( )( )( )+
______________2. *( )( )( +
______________3. *( )( )( )( )+
______________4. {( ) . / ( )( ) }
______________5. {( ) .( ) / ( )}

Remedial Activity 1.2: REPRESENTING FUNCTIONS


DIRECTIONS: Solve the problem and answer the questions below.
Mr. Palma, a Business Math teacher wants his class to run a T-Shirt Printing Business
and agreed to provide up to 50, 000.00 of his own money to help the class get started. To
determine the profitability of the business, the class needs to know the cost to produce the
printed shirts and how many can the class expect to sell for a given price. The class has
identified the following costs:

Heat press machine 24, 480.00


T-shirt 120.00 each
Transfer to press onto each shirt 40.00 each

1. Express the cost C of producing x printed shirts as cost function, C(x)


2. How much does it cost to produce 500 printed t-shirts?
3. How much does it cost to produce 1000 printed t-shirts?

Remedial Activity 1.3: EVALUATING FUNCTIONS


DIRECTIONS: Evaluate the following.
1. Given the function ( ) , find;
a. ( )
b. ( )
c. ( )

2. Given the function ( ) , find;


a. ( )
b. ( )
c. ( )

Remedial Activity 1.4: OPERATIONS ON FUNCTION


DIRECTIONS: Perform the indicated operation.
Given ( ) and ( )
1. ( )( ) ( )( )
2. ( )( ) ( )( )
3. ( )( ) ( )( )
4. ( )( ) ( )( )
5. ( )( ) ( )( )

FOR QUERIES/QUESTIONS, please contact the following teacher/s under your section.
TEACHER STRAND/SECTION CONTACT NO./GMAIL
1) Cherry Cordero STEM A-G 09171156016 rhiechcherry@gmail.com
2) Jerica Nicole Flores HUMSS H-N 09558443974 jrflores1@fatima.edu.ph
3) Jay Vincent Quizon HUMSS A-G 09174340803 jayvincent.quizon@deped.gov.ph
4) Peter Paul Carreon ABM A-G 09773757679 peterpaul.carreon@deped.gov.ph
5) Jessie Manguera ABM H-N, GAS A 09272404388 jessie.manguera@deped.gov.ph
6) Eugene Borbajo GAS B, TVL A-B 09335299090 ecborbajo@yahoo.com

Page 12
APPENDIX A
KEY TO CORRECTION

CHALLENGE 1.1:

 1. f = {(0, -1), (2, -5), (4, -9), (6, -13)}


 2. r = {(a, 0), (b, -1), (c, 0), (d, -1)}
 3. g = {(5, -10), (25, -50), (50, -100)}
NF 4. t = {(-2, 0), (-1, 1), (0, 1), (-2, 2)}
 5. h = { (1,3), (2,6), (3,9), (4,12),… }
F 6.
F 7.
NF 8.

CHALLENGE 1.2:

A tree grows 20cm every year, so the height of the tree is related to its age.

1. Write the function.


h(age)= age · 20 or h(x)=20x

2-3. Find the height of the tree if its age is 5 years.


f(5) = 5(20) = 100cm

Jason goes to Boom-na-Boom theme park where he pays Php 10.00 admission and Php 5.00
per ride.

4. Write a function that describes this situation where f(x) is the total amount of expenses.
f(x) = 5x+10

5-6. Find the amount Jason spent after riding 12 rides.


f(12) = 5(12) + 10 = 70 pesos

CHALLENGE 1.3:
Given: If f(x) = 5x + 2, find

1-2. f(-3) = 5(-3) + 2 = (-15) + 2 = -13

3-4. f(0)= 5(0) + 2 = 0 + 2 = 2

5-6. f(5) = 5(5) + 2 = 25 + 2 = 27

7-8. f( ) = 5( ) + 2 = (-6) + 2 = -4

9-10. f. /= 5. / + 2 = 12.5 +2 = 14.5

11-12. f(a) = 5(a) + 2 = 5a + 2

Page 13
CHALLENGE 1.4:

Given: If f(x) = 3x2 and g(x) = 2x + 1, find

1-2. (f+g)(3) = (3x2) + (2x + 1)


= 3x2 + 2x + 1
= 3(3)2 + 2(3) +1

= 3(9) + 6 + 1
= 27 + 6 + 1
= 34

3-4. (f-g)(3) = (3x2) – (2x + 1)


= 3x2 – 2x – 1
= 3(3)2 – 2(3) – 1

= 3(9) – 6 – 1
= 27 – 6 – 1
= 20

5-6. (fg)(3) = (3x2)● (2x + 1)


= 6x3 + 3x2
= 6(33) + 3(32)

= 6(27) + 3(9)
= 162 + 27
= 189

7-8. ( )(3) =

( )
= ( )

( )
=

9-10. (f ○ g)(x) =f(g(x))


=f (2x + 1)
= 3(2x + 1)2
= 3 [(2x + 1)(2x + 1)]
= 3(4x2 + 2x + 2x + 1)
= 3(4x2 + 4x + 1)
= 12x2 + 12x + 1

11-12. (g ○ f)(2) =g(f(x))


=g (3x2)
= 2(3x2) + 1
= 6x2 + 1
= 6(22) + 1
= 6(4) + 1
= 24 + 1
= 25

Page 14
APPENDIX B
KEY TO CORRECTION
F. DEVELOPING MASTERY

Perform the indicated operation.

Given: f(x) = x 2 − x − 2 g(x) = x +1 , find;

1-2. (f + g)( − 3) = (x 2 − x – 2) + (x +1) 3-4. (f - g)( 3) = (x 2 − x – 2) – (x +1)


= x 2 − x – 2 + x +1 =x2−x–2– x–1
= x 2 − x + x – 2 +1 = x 2 − x – x – 2 –1
= x 2 – 2x – 3
=x2–1
= (-3)2 – 2(3) – 3
= (-3)2 – 1 =9–6–3=0
=9–1=8

5-6. (f g)(4) = (x 2 − x – 2) (x +1) –


7-8. (f/g) (x) =
= x 3 − x 2 – x 2 – x – 2x – 2
( )( )
= x 3 − 2 x 2 – 3x – 2 =
= (4) 3 − 2 (4) 2 – 3(4) – 2 =x–2
= 64 – 2(16) – 12 – 2 = 18

9-10. (f/g) (6) =


– 11-12. (g ○ f) (7) = g(f(x))
( )( )
= g( – )
=
=( – )+1
=x–2 =
= (7)2 – 7 – 1
= 49 – 7 – 1
= 43
Given: f(x) =2x – 4 and g(x) = − 3x + 1, find;

13-14. (f + g)( − 3) = (2x – 4) + (− 3x + 1) 15-16. (f - g)( 3) = (2x – 4) – (− 3x + 1)


= 2x – 4 − 3x + 1 = 2x – 4 + 3x –1
= 2x − 3x – 4 + 1 = 2x + 3x – 4 –1
= −x – 3 = 5x – 5
= −(-3) – 3 = 5(3) – 5
=3–3 = 15 – 5
=0 =10

17-18. (f ○ g) (5) = f(g(x)) 19-20. (g ○ f) (7) = g(f(x))


= f(− 3x + 1) = g(2x – 4)
= 2(− 3x + 1) – 4 = − 3(2x – 4) + 1
= − 6x + 2 – 4 = − 6x + 12 + 1
= − 6x – 2 = − 6x + 13
= − 6(5) – 2 = − 6(7) + 13
= − 32 = −42 + 13
=−29

Page 15
SELF-INSTRUCTIONAL PACKETS
GENERAL MATHEMATICS GRADE 11

September 1 – 4, 2020 (Week 2)


School PAMPANGA HIGH SCHOOL Teaching Dates/ Week
September 7 – 11, 2020 (Week 3)
August 31, 2020 – National Heroes Day (regular holiday) – Monday
Teacher General Mathematics Teachers Quarter First

I. OBJECTIVES

A. Content Standards
The learner demonstrates understanding of key concepts of rational functions.
B. Performance Standards
The learner is able to accurately formulate and solve real-life problems involving
rational functions.

C. Learning Competencies
The learner
1. represents real-life situations using rational functions; (M11GM-Ib-1)
2. distinguishes rational function, rational equation, and rational inequality;
(M11GM-Ib-2)
3. solves rational equations and inequalities; (M11GM-Ib-3)
4. represents a rational function through its: (a) table of values, (b) graph, and (c)
equation; (M11GM-Ib-4)
5. determines the: (a) intercepts, (b) zeros, and (c) asymptotes of rational functions;
(M11GM-Ic-1) and
6. solves problems involving rational functions, equations, and inequalities.
(M11GM-Ic-3)

D. Objectives
At the end of the lesson, the learners should be able to:
1. represent real-life situations using rational functions;
2. distinguish rational function, rational equation, and rational inequality;
3. solve rational equations and inequalities;
4. represent a rational function through its: (a) table of values, (b) graph, and (c)
equation;
5. determine the: (a) intercepts, (b) zeros, and (c) asymptotes of rational functions;
6. solve problems involving rational functions, equations, and inequalities.

II. CONTENT
RATIONAL FUNCTIONS

Learning Resources

A. Reference
 Learner‟s Materials in General Mathematics, pp. 21-58
 Teacher‟s General Guide For General Mathematics, pp. 23-75
B. Other Learning Resources
Our Lady of Fatima University, General Mathematics Online Book, pp. 17-25
https://www.mathworksheets4kids.com/polynomials/recognize/identifying-
polynomials-1.pdf
http://www.mathcentre.ac.uk/resources/uploaded/mc-ty-polynomial-2009-1.pdf

III. PROCEDURES
WEEK 2: Day 1

A. Reviewing previous lesson or presenting the new lesson

Ni Hao! How are you today? Hope poly (nomial), you are doing great! In grade 10
Mathematics, you have covered the concept of polynomial functions.

Page 16
A polynomial function is a function such as a quadratic, a cubic, a quartic, and so on,
involving only non-negative integer powers of x.

DEFINITION: A polynomial is a function of the form

𝒇(𝒙) 𝒂𝒏 𝒙𝒏 𝒂𝒏 𝟏 𝒙𝒏 𝟏
… 𝒂𝟐 𝒙𝟐 𝒂𝟏 𝒙 𝒂𝟎

The degree of a polynomial is the highest power of x in its expression. Constant (non-
zero) polynomials, linear polynomials, quadratics, cubics and quartics are polynomials of
degree 0,1,2,3 and 4 respectively. The function f(x) = 0 is also polynomial, but we say that its
degree is „undefined‟.
Retrieved from: http://www.mathcentre.ac.uk/resources/uploaded/mc-ty-polynomial-2009-1.pdf

Let us start with a short recall about polynomial.

A polynomial is an expression that can have constants, variables and exponents but:

 NO division by a variable / NO variable in denominator


 Exponents of the variables can only be 0, 1, 2, 3,…
Example: 3x2 + 4x + 1
The following are the examples of POLYNOMIALS and NOT POLYNOMIALS.

 3x polynomial
 x–2 polynomial
 √ not polynomial (because of √ )
 polynomial
 not polynomial (because of exponent -2)
 polynomial (exponent zero)
 not polynomial (x has an exponent of -1)
 √ polynomial (exponent zero)
 not polynomial (because of exponent -3)
 polynomial

Is it clear? If yes, let us have a short activity to prepare you what you are going to expect
in this module.

DIRECTIONS: Determine if the following expressions are polynomials or not. Write P if


it is polynomial and NP if it is not. Copy ALL and write the answers on your Gen Math
notebook.
Score Description
______ 1. –3p2q – 4p + 2p7qr + q-4 + 5 5 Naol perfect!
______ 2. y5 – 3y4 + y3 – y
2
______ 3. u – u + 7 + u + u4 7 3-4 Bongga ka!
5
______ 4. –6m n + 2√ 1-2 Pwede na besh
______ 5. 5x2 + + x5 Practice pa more
0
on page 33 sec J

Now, check your work by turning to page 34 for the key to correction. How many correct
answers did you get? Rate your result using the table above. If your score is at least 2 out of 5,
you may now proceed to next part of the discussion.

Page 17
B. Establishing a purpose for the lesson
Let‟s play a game! The title of the game is SEE-PICS ONE WORD.
Identify the word that is being described using the given picture. Write your answers on
the blank provided.

__N __ Q __ __ __ __ __ __
___ ___ ___ A ___ ___ T ___

Did you enjoy the game? Now, check your answer on page 34. The answers in the activity
are the words that you will learn in this lesson.

WEEK 2: Day 2

C. Presenting examples/instances of the new lesson

For you to understand the rational function as representations of real-life situations, here
are the following scenarios to demonstrate real-world applications:

(a) The local barangay received a budget of P100,000 to provide medical checkups
for the children in the barangay. The amount is to be allotted equally among all the
children in the barangay.

Write an equation representing the relationship of the allotted amount per child
(y-variable) versus the total number of children (x-variable).

Answer:

Below is the table with the different allotment amounts for different values for the
number of children using the formula:

No. of children (x) 10 20 50 100 200 300 500 1000


Allocated amount (y) 10,000 5000 2000 1000 500 333 200 100

Now, it is your turn to answer and try the challenge on the next page.

Page 18
DIRECTIONS: Writing rational functions. Solve the following scenarios. Copy ALL and write
the answers on your Gen Math notebook.
Francis Leo Marcos wants to supplement the previous budget allotted by providing an
additional P750 for each child in the barangay.

1. If g(x) represents the new amount allotted for each child, construct a function
representing this relationship.

b. Fill up the table below with the new allotment amounts under this scenario.

No. of children (x) 10 20 50 100 200 300 500 1000


Allocated amount g(x)
2. 3. 4. 5. 6. 7. 8. 9.

Now, check your work by turning to page 34 for the key Score Description
to correction. How many correct answers did you get? If your 9 Naol perfect!
score is at least 4 out of 9, you may now proceed to next part of
5-8 Bongga ka!
the discussion.
3-4 Pwede na besh
WEEK 2: Day 3 Practice pa more
0-2
on page 33 sec J
D. Discussing new concepts and practicing new skills #1

Before we begin, let us differentiate rational expression, rational function, rational


equation, and rational inequality. This will help you to understand the next part of this module.
Lesson 6: Rational Function, Rational Equation, and Rational Inequality
What is a rational function? Are you familiar with the word “rational”?
A rational expression is an expression that can be written as a ratio of two polynomials.
Examples: , , , , 2x- 1, x+5, 3

A rational expression is an expression that can be written as a ratio


of two polynomials.
Rational expression because the numerator 1 is a polynomial (of
degree 0).

Rational expression which is also a polynomial.

√ NOT a rational expression since the numerator is not a polynomial.

The expression is equal to ( )( )


so it is a rational expression.

You may use the table on the next page to show how to distinguish among rational
equations, rational inequalities and rational functions.

Rational Function Rational Equation Rational Inequality


It is a function which is the
ratio of polynomial
functions. An inequality involving
In symbol; An equation rational expressions.
Definition ( )
f(x) = ( ) involving rational
expressions. It uses symbols like ≥,
where in: p and q are ≤, >, or < sign.
polynomial functions and
( )

Page 19
Rational Function Rational Equation Rational Inequality

1. 1. 1.
Examples 2. ( ) 2. 2.
3. ( ) 3.

DIRECTIONS: Determine whether the given is rational function, rational equation, rational
inequality, or none of these. Copy ALL and write the answers on your Gen Math notebook.

1. ___________________________________
2. ___________________________________
3. √ =5 ___________________________________
4. ___________________________________
5. 6x ≥ ___________________________________

6. ___________________________________
7. ___________________________________
8. ( ) ___________________________________
9. ___________________________________
10. ___________________________________

Score Description
Now, check your work by turning to page 34 for the key
to correction. How many correct answers did you get? Rate the 10 Naol perfect!
result using the table on the right. If your score is at least 5 out 6-9 Bongga ka!
of 10, you may now proceed to next part of the discussion. 4-5 Pwede na besh
Practice pa more
0-3
WEEK 2: Day 4 on page 33 sec J

E. Discussing new concepts and practicing new skills #2

Are you now ready to solve? Let us now start the solving proper! We need your skills
in solving a linear and quadratic equation.

Lesson 7.1: Rational Equations

To solve an equation or inequality in one variable such as x means all values of x should
satisfy the equation or inequalities so that the equation or inequality is true.

Procedure for Solving Rational Equations

To solve rational equations:


1. Eliminate the denominator by multiplying both sides by the Least Common Denominator
(LCD).
2. Solve for the value of x by equating to zero, by factoring, by completing square or by
using quadratic formula.
3. Check your solution by substituting the obtained values of x to the original equation. If
both equation on the left and right are equal, then your answer is correct, if not, check for
extraneous root or go back to your previous solution.

Page 20
Illustrative Example 1:

Solve for x:

Solution: Checking:
The LCD of all denominators is 10x. Multiply Since the solution to the equation is , then
both sides of the equation by 10x and solve
the resulting equation. substituting x = in the original equation must
make this equation true.
( ). / ( ). / ( ). /

20 – 15 = 2x

5 = 2x . /

or (True)

Therefore, is the solution.

Illustrative Example 2:
Solve for x:
Solution:
Factor each denominator in the fraction. Since is factorable by( )( ), the
equation will become:
( )(
.
)
The LCD is ( )( )
Multiply the LCD to both sides of the equation to remove the denominators.

( )( ) ( )( ) = [( )( )] . /
( )( )
( ) ( )

Upon reaching this step, we can use strategies for solving polynomial equations (by
factoring).

( )( ) Factoring
( ) ( ) Zero Property
Roots

Let us check the answer by substituting the value of x.

If x = 5, If x =– 2,

- = - =

- = - =

- = - =

- = Since the denominator is zero, it makes the


equation undefined. Therefore, -2 is called
extraneous root and it is not a solution to the
(True)
equation.

Therefore, the only solution in the equation is 5.

Page 21
DIRECTIONS: Solve the equation by finding the value of x and check the answer.
Copy ALL and write your answers and solution on your Gen Math notebook. (5 points
each)

1. = 3.

2. 4. =

Points Rubrics
1 There is a solution but no answer
2 Correct solution but the answer is wrong.
3 Correct solution and answer but no checking.
4 Correct solution and answer but there is missing part in checking.
5 Correct solution, answer and there is checking.

Score Description
Now, check your work by turning to page 34 for the key 20 Naol perfect!
to correction. See the rubrics above. How many correct 11-19 Bongga ka!
answers did you get? Rate your result using the table on the
right. If your score is at least 10 out of 20, you may proceed to 5-10 Pwede na besh
next part of the discussion. If not, ask the teacher. Practice pa more
0-4
For the next part of the discussion, we are going to on page 33 sec J
solve rational inequalities.

WEEK 3: Day 1

Lesson 7.2: Rational Inequalities


Procedure for Solving Rational Inequality
To solve rational inequalities:
1. Rewrite the inequality as a single rational expression on the left side and zero on the
right side. (All rational expression on the left and zero on the right. Simplify the left side
before proceeding to the next step.)
2. Determine the meaningful numbers by setting the numerator equal to zero and the
denominator equal to zero. (Solve the value of x both numerator and denominator that
makes the equation 0.)
3. Use number line to graph the value of x. If the symbol of inequality is , both value of
x on the numerator is not included, use open circle. If the symbol of inequality is
the value of x on the numerator is included, use solid circle and the value of x on the
denominator is always not included (open circle).
4. Separate the number line into intervals. (See the table on the next page)
5. Select and test a value in each interval. (Select a value easy to substitute in between
the interval)
6. The intervals that satisfy the inequality are the answer.

To simplify the process remember key words: Single Rational Expression, Meaningful
Numbers, Graph, Intervals, Test value, Answer. (SiREx MeN GITA)

Table of Interval Notation


The set notation * + is read as “set of x such that x is greater than a but less
than b” or “set of x such that x is between a to b”.

Page 22
Examples:
1. The set notation * + means the solution is between 2 to 5, both 2 and 5 are
not included. The interval notation is (2, 5).
2. The set notation * + means the solution is between 2 to 5, 2 is included and 5
is not included. The interval notation is [2, 5).
3. The set notation * + means the solution is between 2 to 5, 2 is not included
and 5 is included. The interval notation is (2, 5].
4. The set notation * + means the solution is between 2 to 5, both 2 and 5 are
included. The interval notation is [2, 5].
5. The set notation * + means the solution is greater than 5 and 5 is not included.
The interval notation is (5, ∞).
6. The set notation * + means the solution is greater than or equal to 5 and 5 is
included. The interval notation is [5, ∞).
7. The set notation * + means the solution is less than 5 and 5 is not included. The
interval notation is (-∞,5).
8. The set notation * + means the solution is less than or equal to 5 and 5 is
included. The interval notation is (-∞, 5).
Note: Use parenthesis symbol ( , ) if the value is not included in the solution.
The infinity symbol always used ( , ) since the exact value of the number is not defined.
Use bracket symbol [ , ] if the value is included in the solution.

WEEK 3: Day 2

Illustrative Example 1: Solve


STEP 1: Single rational expression
Subtraction Property of Equality
Express the left side as one expression by finding the LCD
Single rational inequality

STEP 2: Meaningful Numbers


If not to test, use this shortcut.
Numerator Test
X = ½ numerator (Since the inequality is >,
therefore ½ is not included.
X = 0 denominator (always not included since it
False, x =1/2 is not a makes the inequality undefined.
solution

undefined
False, x = 0 is not a
solution.

Page 23
STEP 3: Graph

STEP 4: Interval

(- ∞ , 0) (0, 1/2) (1/2,∞)

STEP 5: Test Values

Test (- ∞ , 0) (0, 1/2) (1/2,∞)


Values X = -1 X=¼ X=1

>2 >2 >2


-1 >2 1 >2
4>2
False False
True

STEP 6: Answer/Solution Set


Since the interval (0, ½) makes the inequality true, then the solution is (0, ½).
Shade the graph from 0 to ½ (go back to step 3).

Illustrative Example 2:

Solve

STEP 1: Single rational expression

Subtraction Property of Equality

Simplify
, ( )-
Express the left side as one expression by finding the LCD

Perform the indicated operation

Single rational expression

STEP 2: Meaningful Numbers

Subtraction Property of Equality (SPE)


Division Property of Equality (DPE)
(The meaningful number in numerator is 10, since the inequality is ≤,
-10 is included.)

Addition Property of Equality (APE)


(The meaningful number in denominator is 5, and 5 is not included.)

STEP 3: Graph

STEP 4: Interval
(- ∞ , 5) (5, 10] [10,∞)

Page 24
STEP 5: Test Values

Test Values (- ∞ , 5) (5, 10] [10,∞)


X=0 X= 6 X= 11

( ) ( ) ( )

1
7
True
False
True

STEP 6: Answer/Solution Set


Since the interval (-∞, 5) and [10, ∞) makes the inequality true, then the solution is (-∞,
5) and [10, ∞). Shade now the graph from -∞ to 5 and 10 to ∞ (go back to step 3).

DIRECTIONS: Solve the given inequality. Show your solution following steps 1 to 6.
Copy ALL and write the answers and solutions on your Gen Math notebook.

1. 2.

Process Points Rubrics


2 Complete solution for single rational expression
STEP 1
1 Incomplete solution
2 Correct and complete value(s) for x
STEP 2 1 Correct but incomplete value(s) for x
0 No answer and solution
2 Correct and complete graph for the value(s) for x
STEP 3 1 With answer but incomplete graph for the value(s) for x
0 No answer and solution
2 Complete intervals
STEP 4 1 Incomplete and incorrect intervals
0 No answer and solution
3 Complete test values and correct answer in the substitution
2 Incomplete test values but correct answer in the substitution
STEP 5
1 Incomplete test values and incorrect answer in the substitution
0 No answer and solution
2 Complete answer
STEP 6
1 Incomplete answer
0 No answer and solution

Page 25
Now, check your work by turning to page 36 for the key Score Description
to correction. See the rubrics on the right. How many correct 20 Naol perfect!
answers did you get? Rate your result using the table on the
11-19 Bongga ka!
right. If your score is at least 10 out of 20, you may proceed to
next part of the discussion. If not, ask the teacher. 5-10 Pwede na besh
Practice pa more
0-4
on page 33 sec J
WEEK 3: Day 3

Lesson 8: Rational Function Through its: Table of Values, Graph and Equation

Representation of function can be in the form of equation or function notation, table of


values and graphs.

Illustrative Examples:

2
Given: y 
x 2
(a) Equation (b) Table of Values
2 X
y
x 2
As we assign values 1.5 -4
Consider the graph of the function for x that are smaller
2 but closer to 2, the 1.7 -6.67
y , this function has one restricted
x 2 value of y gets
closer and closer to 1.9 -20
value that generate zero on denominator, that
is x=2. Since x=2 cannot use for x y table, negative infinity.
2.0 ±∞
however it does not mean that we cannot use
x that is close to 2. 2.1 20
As we consider
2.3 6.67
values of x that are
larger than 2 but 2.5 4
closer to 2, the value
of y gets closer and
closer to positive infinity.

(c) graph

Looking at the graph, as x


values get larger and larger in
negative direction, the y-values of
the graph appear to get closer and
closer to zero.

Observe that the function


will be undefined at x = 2. This
means that there cannot be a line
passes through x = 2, instead use
dotted vertical line.

 To represent a rational function using table of values, get any value of x (do not make
the function undefined) and substitute it to the function to get the value of the function x
or f(x) and present it in a table vertically or horizontally.

Page 26
 To represent a rational function using graph, use graphing application like desmos,
geogebra, mathway, and malmath. You may download it using your phone, tablet or
laptop.

GEOGEBRA MATHWAY MALMATH DESMOS

Do you still remember how to plot points in Cartesian plane? We used ordered pairs
(x,y) in plotting points. In this lesson, I will show you different graphs of rational function and
their asymptotes.
WEEK 3: Day 4

Lesson 9: Intercepts, Zeros, and Asymptotes of Rational Function

 Intercepts
Intercepts are x or y coordinates of the point at which a graph crosses the x-axis or y-axis.

1. x – intercept
It is the x-coordinate of point (x,0) where the graph crosses the x-axis. To find x-
intercept, substitute 0 for y and solve for x.
2. y –intercept
It is the y-coordinate of point (0,y) where the graph crosses the y-axis. To find y-
intercept, substitute 0 for x and solve for y.

Illustrative Example 1: ( )

x-intercepts : y-intercepts :
Set y = 0 Set x = 0
( ) ( )
Change f(x) to y
( ) Substitute x to 0
Cross multiply

0-8 = x+ 8 - 8 SPE y = -4 y-intercept


-8 = x x-intercept
(0,-4) ordered pair
(-8, 0) ordered pair

Illustrative Example 2: ( )

x-intercepts: y-intercepts:
Set y = 0 Set x = 0
( ) ( )
Change f(x) to y ( )
( ) Substitute x to 0
Cross multiply
DPE
0=x x-intercept y=0 y-intercept
(0,0) ordered pair
(0,0) ordered pair

Page 27
 Zeros of Rational Functions

The values of x that will make the numerator zero without simultaneously making the
denominator equal to zero. Zeros of a function is also the x-intercept.
Finding the zeros of rational functions:
1. Factor the numerator and the denominator and cancel common factor(s).
2. Identify the x-values that will make the numerator equal to zero.
3. Identify the zeros.

*Hole(s) is (are) produced when numerator and denominator have common factor(s.)

Illustrative Examples:

1.) Consider the function ( ) , find the zero of a function.

The numerator x - 2 will be zero at x = 2. Therefore x = 2 is a zero of f(x).


( )( )
2.) Given: ( ) = ( )( )
, the numerator x + 4 will be zero at x = – 4.

Therefore x = – 4 is a zero of f(x). The hole is – 2 produced by the common factor x + 2, equate
it to zero and solve for x.
( )( )
3.) Given: ( ) =( )( )

The numerator x – 2 will be zero at x = 2. Therefore x = 2 is a zero of f(x). There is a hole which
is 2 produced by the factor x – 2, equate it to zero and solve for x.
 Asymptotes of Rational Functions

Consider the situation in basketball court, as an audience, if you want to see the game
nearly, you move closer and closer to the court but not allowed to enter inside the court.
This idea is similar to the definition of asymptote.
Asymptote is a line that the graph of function gets close to but does not touch the graph.
There are three different asymptotes:

1) Vertical Asymptote
- restrictions on the x-values of a reduced rational function
- to find the restrictions, equate the denominator to 0 and solve for x
- common factors canceled on both N(X) and D(X) represent holes in the graph.
2) Horizontal Asymptotes
Let n be the degree of numerator and m be the degree of denominator.
- If n < m, the horizontal asymptote is y = 0.
- If n = m, the horizontal asymptote is y = a/b.
where: a – leading coefficient of N(x)
b – leading coefficient of D(x)
- If n > m, the graph has no horizontal asymptote.
3) Oblique or Slant Asymptotes
- Occurs when the numerator of f(x) has a degree that is one higher than the degree of
the denominator.
- If n > m, the asymptote will be oblique or slant (y = mx + b).
- To get the equation of the oblique asymptote, simply divide the numerator by the
denominator by either using long division or synthetic division.
- The oblique asymptote is the quotient with the remainder ignored and set equal to y.

Page 28
Illustrative Examples:
Function Vertical Asymptote Horizontal Asymptote
Degree of numerator (n) is 0 since whole
x + 1 = 0 Equate the denominator
( ) number
1. to 0
Degree of denominator (m)
x = –1 Solve for x by SPE
is 1.Then, n < m, therefore y = 0.
The vertical asymptote is x = –1.
The horizontal asymptote is y = 0.
Degree of numerator (n) is 1,
2x + 1=0 Equate the denominator Degree of denominator (m)
( ) to 0.
2. is 1.Then, n= m, therefore y = = .
x = – Solve for x by SPE and DPE
4 - leading coefficient of numerator
The vertical asymptote is x = – . 2 - leading coefficient of denominator.
The horizontal asymptote is y = 2.
Degree of numerator (n) is 2
Degree of denominator (m)
x-1=0 Equate the denominator to 0 is 1.Then, n>m, therefore there is no
3. ( )
x = 1 Solve for x by APE horizontal asymptote, instead it is an
The vertical asymptote is x = 1. oblique asymptote.
(no need to divide the function and get
the equation).

DIRECTIONS: Determine the intercepts and asymptotes of the following by completing the
table. Copy ALL and write the answers on your Gen Math notebook. (1 point each)

Function X- y-intercept Zeros and Vertical Horizontal


intercept (x=0) Hole(s) Asymptote Asymptote
(y=0)
1. ( ) Y= 0

2. ( ) =
( )( )

There is no
zeros of the
function.
The hole is
5.
3. ( ) None, Since
it oblique.

4. ( ) None

5. ( ) X=1

Now, check your work by turning to page 37 for the Score Description
key to correction. How many correct answers did you get? 20 Naol perfect!
Rate your result using the table on the right. If your score is at 11-19 Bongga ka!
least 10 out of 20, you may proceed to next part of the 5-10 Pwede na besh
discussion. Practice pa more
0-4
on page 33 sec J

Page 29
WEEK 3: Day 5

Lesson 10: Solving Problems involving Rational Functions, Equations, and Inequalities

Problem 1. An airforce plane flies at a speed that is 30 kilometers per hour faster than a cargo
plane. The air force plane flies 500km in the amount of time that the cargo plane flies 440km.
Find the speed of each plane.

Let: r = the speed of the cargo plane in kph


r + 30 = the speed of the air force plane in kph
Since distance = rate x time, we can fill up the table as follows.

Plane D R t=

Air Force 500 r + 30

Cargo 440 R

Solution:

Since the time of travel for the air force plane is equal to the time of the cargo
plane, the equation should be written as:

t(air) = t(car) Form the equation


( ) ( )
( ) ( )
Substitute the formula for time which is t=d/r for

both sides of the equation


Substitute the values of d and r from the table

( )( ) ( )( ) Multiply each term to the LCD

500r = 440r + 13 200 Cancel common factors


60r = 13 200 Simplify by combining similar terms
r = 13200/60 Divide both sides by 60 (DPE)
r = 200 This is the rate of the cargo plane

Since the rate of the air force plane is 30kph faster than the cargo plane, then
r(air) = 220 + 30 = 250

Therefore, the cargo plane travels at 220kph while the air force plane travels at
250kph.

Problem 2: Maria can paint a kitchen in 3 hours and James can paint the same kitchen in 2
hours. How long would it take for both working together to paint the kitchen? Use the formula
rate = amount of work / time (r = w / t).

Given: Maria can paint the whole kitchen in 3 hours, then she can do 1/3 of the job in 1 hour
rate of Maria = 1/3 kitchen per hour
James can paint the kitchen in 2 hours, then he can do 1/2 of the job in 1 hour.
rate of James = 1/2 kitchen per hour
Let x= the number of hours it would take both to paint the kitchen. Their combined rate
would be, rate of both Maria and James = 1/x

Page 30
Solution:

Combined rate equation:


rate of Maria + rate of James = rate of both

Since rate = work/time, express each rate as a ratio

( ) ( ) ( ) Multiply the LCD to each term to remove the denominator

Simplify the equation


Combine similar terms
Divide both sides by 5 (DPE)
Therefore, it would take Maria and James hours or 0.83 hours to paint the kitchen together.

F. Developing mastery
GENERAL DIRECTIONS: Copy ALL and write the answers on your Gen Math notebook.
 Solve rational equation and check the answer.

1.

 Solve rational inequality and show your solutions by following steps 1 to 6.

2.

Now, check your work by turning to page 38 for the key to correction.

G. Finding practical applications of concepts and skills in daily living

1. Distance, time and speed. Distance is the product of speed and time. These units are
related by the following formula:
d= rt , r = , t =

2. Units of temperature, Celsius(C ), Fahrenheit(F), and Kelvin(K).These units are


related by the following formula:
C= ,C= , and K = C + 273.

3. The number of diagonals (d) of a polygon of n sides is given by d = ( )

4. The net resistance ( ) of an electrical circuit containing resistors in a


parallel is given by = .

H. Making generalizations and abstractions about the lesson

 What is the difference between rational equations to rational inequalities?

Rational equation involves the equality of two or more rational expression that uses =
symbol.
Rational inequality involves the inequality of two or more rational expression that uses ≥, ≥,
≤, >, or <symbol.

 What is the difference between rational equations to rational function?

Rational function is a function that the value of the f (x) depends on the input value of x. It
uses two different variable which is dependent and independent variable.
Rational equation is equality between rational expressions and it uses only one variable.

Page 31
I. Evaluating learning
GENERAL DIRECTIONS: This part is recorded and graded. Copy ALL the given using this
format on a separate yellow sheet of paper. The yellow paper will be submitted to your Math
teachers on the day of the scheduled retrieval.

ACTIVITY 2.1 IN GENERAL MATHEMATICS


(WEEK 2)
Name: _________________Section: _________ Parent‟s Signature: ___________________
RATIONAL FUNCTION, RATIONAL EQUATION AND RATIONAL INEQUALITY
DIRECTIONS: Identify the following if it is rational function, rational equation, rational inequality,
rational expression or none of these.

1. ( )= 6. =
2. = 7. > f(x)
3. ≤ 8.
4. =y 9. =
5. ( ) 10. =
DIRECTIONS: In your own word, how will you differentiate rational function to rational equation
and rational equation to rational inequality? Note: minimum of 5 sentences.
____________________________________________________________________________
____________________________________________________________________________
____________________________________________________________________________

ACTIVITY 2.2 IN GENERAL MATHEMATICS


(WEEK 2)
Name: _________________Section: _________ Parent‟s Signature: ___________________
SOLVING RATIONAL EQUATION AND INEQUALITIES
DIRECTIONS: Solve the given rational equation and check the answer. Show your solution.
1. 3.
2. 4.

ACTIVITY 3.1 IN GENERAL MATHEMATICS


(WEEK 3)
Name: _________________Section: _________ Parent‟s Signature: ___________________
DIRECTIONS: Solve the following rational inequalities following the 6-steps.
1. 2. 3.

ACTIVITY 3.2 IN GENERAL MATHEMATICS


(WEEK 3)
Name: _________________Section: _________ Parent‟s Signature: ___________________
RATIONAL FUNCTION
DIRECTIONS: Complete the table below. Determine the intercepts, zeros and asymptotes of
each rational function.
X-INTERCEPT Y-INTERCEPT ZEROS OF THE VERTICAL HORIZONTAL
FUNCTION
(y = 0) (x= 0) FUNCTION/HOLE ASYMPTOTE ASYMPTOTE

1. ( )

2. ( )

3. ( )

4. ( )

5. ( )

Page 32
J. Additional activities for application or remediation
GENERAL DIRECTIONS: Copy ALL and write your answers on your Gen Math notebook.

Remedial Activity 2.1: POLYNOMIALS


DIRECTIONS: Determine if the following expressions are polynomials or not. Write P if it is
polynomial and NP if it is not.
______ 1. ______ 3. √ ______ 5. +
______ 2. – 5c2 + c – 9 ______ 4. 7m4 + 3√ +2
Remedial Activity 2.2: REPRESENTING RATIONAL EXPRESIONS
Carlos lives about 20 kilometers from office. For one week, Carlos keeps track of time it
takes him to get to the office as shown in the table. He wanted to figure out his average speed
based on the time recorded. Can you help Carlos?
Day Time (hours) Speed (kph)
Monday 0.5
Tuesday 0.6
Wednesday 0.75
Thursday 0.7
Friday 1
Any day N
1. Write a formula relating speed, time, and distance.
2. Use the formula in (a) to determine Carlos‟ average speed each day. Record the results
in the last column of the table.
3. If t is the time and r is the average speed, write the equation for the function that
contains Carlos‟ average speed when the input is time.
Remedial Activity 2.3: RATIONAL EXPRESSIONS
DIRECTIONS: Identify the following if it is rational function, rational equation, rational inequality,
rational expression or none of these.
1. ( ) 6.
2. 7.
3. 8.
4. ( ) 9.

5. ( ) 10. ( )
Remedial Activity 2.4: SOLVING RATIONAL EQUATIONS
DIRECTIONS: Solve the following rational equations and check the answer.
1. 3.
2. 4.
Remedial Activity 3.1: SOLVING RATIONAL INEQUALITIES
DIRECTIONS: Solve the following rational inequalities following the 6 steps.
1. 2.
Remedial Activity 3.2: RATIONAL FUNCTIONS
DIRECTIONS: Complete the table below. Determine the intercepts, zeros and asymptotes of
each rational function.
X-INTERCEPT Y-INTERCEPT ZEROS OF THE VERTICAL HORIZONTAL
FUNCTION
(y = 0) (x= 0) FUNCTION/HOLE ASYMPTOTE ASYMPTOTE

1. ( )

2. ( )

3. ( )

4. ( )

5. ( )

Page 33
APPENDIX A
KEY TO CORRECTION

CHALLENGE 2.1:

NP 1. –3p2q – 4p + 2p7qr + q-4 + 5 NP 4. –6m5n + 2√


P 2. y5 – 3y4 + y3 – y NP 5. 5x2 + + x5
P 3. u – u2 + 7 + u4 + u7

B. Establishing a purpose for the lesson


1. Asymptote
2. Intercept
3. Equality
4. Inequality.

CHALLENGE 2.2:

1.) A Francis Leo Marcos wants to supplement the previous budget allotted by providing an
additional P750 for each child in the barangay.

a. If g(x) represents the new amount allotted for each child, construct a function
representing this relationship.
b. Fill up the table below with the new allotment amounts under this scenario.

No. of children (x) 10 20 50 100 200 300 500 1000

Allocated amount 10 750 5 750 2 750 1 750 1 250 1 083.33 950 850
g(x)

CHALLENGE 2.3:

1. Rational Function 6. None Of These


2. Rational Equation 7. Rational Equation
3. None of these 8. Rational Function
4. Rational Equation 9. Rational Inequality
5. Rational Inequality 10. Rational Equation

CHALLENGE 2.4:
Check: If x = 5/6,
1.

( ) ( ) ( )
𝑥 𝑥

Therefore, the solution is 5/6.

Page 34
2. Check: If x = 5/2,

( )( ) ( )( )

Therefore, the solution is 5/2.

Check: If x = - 5/9,
3.
. /
( )( ) ( )( ) ( )( ) . /
( ) ( ) ( )( )
( )

𝑥 𝑥

Therefore, the solution is – 5/9.

4. Check:
If x = - 1,
( )
( ) (x+1)

=1

-1 = 0

(x+1)(x-1)=0 (Undefined)

x+1=0 x-1=0
If x = 1,
x=-1 x=1 ( )

The equation is undefined at x = -1 therefore, it as an


extraneous root. The only solution is 1.

Page 35
CHALLENGE 3.1:

1.

Step 1:

Single rational expression

Numerator Denominator
1 – 4x = 0 x=0
not included
Step 2:

not included

Step 3:
0 1/4

Step 4:
( -α , 0 ) ( 0, ) ( ,α)
( -α , 0 ) ( 0, ) ( ,α)
x = -1 x=1

Step 5:
1<4
True True
8<4
False

0 1/4
Step 6:
The interval ( -α , 0 ) and ( , α ) makes the inequality true. Therefore the solution
is ( -α , 0 ) U ( , α )

2.

Single rational expression


Step 1:

Numera or Denominator
x+2=0 x–5=0
Step 2: x = -2 x=5
included not included

Page 36
Step 3:
-2 5

Step 4:
( -α , -2 ] [ -2, 5 ) ( 5, α )

( -α , -2 ] [ -2, 5 ) ( 5, α )
x = -3 x=0 x=6
Step 5:

False True False

Step 6:
Therefore the solution set is [ -2, 5 )

CHALLENGE 3.2:

Function X- y-intercept Zeros and Vertical Horizontal


intercept (x=0) Hole(s) Asymptote Asymptote
(y=0)
1. ( ) x=0 y=0 x=0 X=4 y =0

2. ( ) x=5 y= 1/5 = X = ±5 y=0


( )( )

There is no
zeros of the
function. The
hole is 5.

3. ( ) x= -10, x= x= -10, x= 2 X = -3 None, Since


2 No hole it oblique.
4. ( ) None None None X = -7 y=0

5. ( ) x=2 x=2 X=-6 X=1

Page 37
APPENDIX B
KEY TO CORRECTION
F. Developing mastery
Solve rational equation and check the answer.

( )( ) ( )( ) Multiply each term by the LCD


( ) = 2(x+1) Distributive Property
Simplify by combining similar terms
3x-2x = 9+2 Simplify
x=11
Check: x = 11

= True
Solve rational inequality and follow the 6-step.

Step 1: Single rational Expression

Step 2: Value(s) of x
Numerator Denominator

x+1=0 x+3 = 0

x= -1 x= -3

Included not included

Step 3: Graph

-3 -1
Step 4: Interval
(- -3) (-3,-1] [- )
Step 5: Test values

Interval (- -3) (-3,-1] [- )

Test value x= -4 x= -2 x= 0

- False

False True

Step 6: The solution set is the interval (-3,-1] that makes the inequality true.

-3 -1

Page 38
SELF-INSTRUCTIONAL PACKETS
GENERAL MATHEMATICS GRADE 11

School Teaching Dates/ Week


September 14 – 18, 2020
PAMPANGA HIGH SCHOOL (Week 4)
Teacher GENERAL MATHEMATICS TEACHERS Quarter First

I. OBJECTIVES

A. Content Standards
The learner demonstrates understanding of key concepts of inverse functions.

B. Performance Standards
The learner is able to apply the concepts of inverse functions to formulate and solve
real-life problems with precision and accuracy.

C. Learning Competencies
The learner
1. represents real-life situations using one-to one functions; (M11GM-Id-1)
2. determines the inverse of a one-to-one function; (M11GM-Id-2)
3. represents an inverse function through its: (a) table of values, and (b) graph;
(M11GM-Id-3) and
4. solves problems involving inverse functions. (M11GM-Ie-2)

D. Objectives
At the end of the lesson, the learners should be able to:
1. represent real-life situations using one-to one functions;
2. determine the inverse of a one-to-one function;
3. represent an inverse function through its: (a) table of values, and (b) graph; and
4. show appreciation to the topic by relating and solving real –life situations.

II. CONTENT
INVERSE FUNCTIONS

Learning Resources

A. Reference
General Mathematics Learner‟s Material pages 60-64
General Mathematics for Senior High School - Baningon et.al pages 49-53

III. PROCEDURES
WEEK 4: Day 1

A. Reviewing previous lesson or presenting the new lesson

Aloha! As we start of our lesson, let us recall about the one-to-one relations by answering
the challenge below.

DIRECTIONS: Determine if the following relation represents ONE-TO-ONE relations or


not. Copy ALL and write the answers on your Gen Math notebook.

__________ 1. M = {(3, -1), (2, -1), (1, -1), (0, -1)}


__________ 2. A = {(0, 0), (2, 1), (4, 2), (6, 3), …}
__________ 3. The relation pairing a PHS student to his or her student number.
__________ 4. The relation pairing an airport to its airport code
__________ 5. The relation pairing a real number to its square.

Page 39
__________6. X Y
h Score Description
11 6 Naol perfect!
7 13 4-5 Bongga ka!
17 3 Pwede na besh
2 19 Practice pa more
23 0-2
on page 48 sec J

Now, check your work by turning to page 49 for the key to correction. How many correct
answers did you get? Rate your result using the table above. If your score is at least 3 out of 6,
you may now proceed to next part of the discussion.

In this module, you will learn more about one-to-one functions and its inverse function.
You will also solve problems involving inverse functions. Let‟s get ready!

B. Establishing a purpose for the lesson

REFLECT:

Is there someone you know that is opposite to your


personality? In science, there is a concept known as “opposite
attracts”. Can you find your exact inverse in real-life?

Images retrieved from:


 https://www.google.com/search?q=mirror+person+icon&sxsrf=ALeKk03R61fRYuWmMd0wlRMXtNc5GY4NEQ:1591515060763&source=lnms&tbm=isc h&sa=X&v
ed=2ahUKEwi_vOfOl-_pAhWe7XMBHcOZAyQQ_AUoAXoECAsQAw#imgrc=K_Fm2DQtc6cLoM

C. Presenting examples/instances of the new lesson


Lesson 11: One-to-one Functions

REMEMBER ME!
Definition of a one-to-one function:
A one-to-one function is a function in which every element in the range of a function
corresponds to exactly one element in the domain.

In Illustrative Examples 1-4, let us determine whether the given relation is a function. If it
is a function, let us determine whether it is one-to-one or not.

Illustrative Example 1: M = {(3, -1), (2, -1), (1, -1), (0, -1)}
Since the domains for this function are different
from each other, we can say that this is a function. However, since there are y-values that
have more than one x-value, this is said to be many-to-one function.

Illustrative Example 2: The relation pairing a PHS student to his or her student number.
Each PHS student is assigned to a unique student number. Thus, the relation is a
function. Further, two different students cannot be assigned the same student number. Thus,
the function is one-to-one function.

Illustrative Example 3: The relation pairing a real number to its cube.


Each real number has a unique perfect cube. Thus, the relation is a function. However,
two different real numbers such as 2 and –2 may have the same cube. Thus, the function is
not one-to-one.

Page 40
Illustrative Example 4: Observe the following graphs. Which of them are functions?
Which of them are one-to-one functions?

A B C

D E

A simple way to determine if a given graph is that of a function is by using the Vertical
Line Test.

Vertical Line Test – A relation is a function if each vertical line does not intersect the graph at
more than one point.

A As you can see, if we will use a D


vertical line test in graph A, the vertical
line intersect the graph at two points.
Therefore, this graph is a mere relation.

In graphs B, C, D and E, the


vertical line will only intersect at exactly
one point. These graphs are considered
functions.

In addition, to determine if the graph is that of a one-to-one function, we will use the
Horizontal Line Test.

Horizontal Line Test – A function is a one-to-one if each horizontal line does not intersect the
graph at more than one point.

Observe graphs B, C, D and E, which of these functions are considered one-to-one


functions?

As you can see, the horizontal C


D intersects the graph D at two points which
makes the function as not a one-to-one
function. In the same way, graph E is also
not a one-to-one function.

But if we apply it on graphs B and


C, we will determine that they are one-to-
one functions because the horizontal line
intersects at exactly one point.

Do you know now how to use vertical line test and horizontal line test? A piece of cake,
right? So, it‟s activity time!

Page 41
DIRECTIONS: Put a check mark on the blank provided before each number if each relation
represents a one-to-one function. Copy ALL and write the answers on your Gen Math
notebook.
__________ 1). True or false questions to answers
__________ 2). While reading your textbook, you find a function that has two inputs that
produce the same answer. Score Description
__________ 3). The length of a rectangle is four more than
5 Naol perfect!
its width. Let be the function mapping the
length of the rectangle to its area. 4 Bongga ka!
__________ 4). Consider the uppercase letter of English 3 Pwede na besh
Alphabet S as a graph. Practice pa more
0-2
__________ 5). y = x2 + 4x -3 on page 49 sec J

Now, check your work by turning to page 49 for the key to correction. How many correct
answers did you get? Rate your result using the table above. If your score is at least 3 out of 5,
you may now proceed to next part of the discussion.

D. Discussing new concepts and practicing new skills #1

WEEK 4: Day 2

The importance of one-to-one functions is due to the fact that these are the only
functions in which their inverse functions exist, as explained in this part of the lesson.
Lesson 12: Inverse of a One-to-One Functions

In finding the inverse of the function, we will follow the steps below:
(Step 1) Write the function in the form y = f(x);
(Step 2) Interchange the x and y variables;
(Step 3) Solve for new y in terms of x
(Step 4) Replace the new y with f-1(x).

Illustrative Example 1: Find the inverse of f(x) = 5x – 8

Solutions: f(x) = 5x – 8 Given


y = 5x – 8 Step 1: Write the function in the form y = f(x)
x = 5y – 8 Step 2: Interchange the x and y variables
x + 8 = 5y Step 3: Solve for the new y
(Addition Property of Equality)
Multiplication Property of Equality
Symmetric Property
( ) Step 4: Replace the new y with ( )

Final Answer: The inverse of f(x) = 5x – 8 is ( ) .

Illustrative Example 2: Find the inverse of g(x) =


Solutions: g(x) = Given
y= Step 1: Write the function in the form y = g(x)
x= Step 2: Interchange the x and y variables
Step 3: Solve for the new y
(Addition Property of Equality)
√ Take the cube root both side of both sides
( ) √ Step 4: Replace the new y with ( )

Final Answer: The inverse of g(x) = is ( ) √ .

Page 42
Illustrative Example 3: Find the inverse of rational function of ( ) .

Solutions: ( ) Given
Step 1: Write the function in the form y = h(x)
Step 2: Interchange the x and y variables
( ) Step 3: Solve for the new y
(Cross Multiplication)
Distributive Property of Multiplication over addition
Putting all terms with y on the left side and those
without y on the right side
(Addition Property of Equality)
( ) Common Monomial Factor
Dividing both sides by 3x-2
( ) Step 4: Replace the new y with ( )
Final Answer: The inverse of rational function of ( ) is ( ) .

Illustrative Example 4: Find the inverse of ( )

If you will notice, the degree (highest exponent) of this


equation is 2 which makes it a quadratic equation. Furthermore,
the shape of this graph for this equation is a parabola which opens
upward. Using a horizontal line test, this shows that it is not a one
to one function.

REMEMBER ME!
A function has an inverse if and only if it is one-to-one function.

Therefore, the function ( ) has no inverse function.

To add more details about inverse functions, here are their properties. Always keep
them in your mind!

DO NOT FORGET!
PROPERTIES OF AN INVERSE OF A ONE-TO-ONE FUNCTION
Given a one-to-one function ( ) and its inverse ( ), then the following are true:
 The inverse of ( ) is ( )
 ( ( )) for all x in the domain of
 ( ( )) for all x in the domain of f

Let us check if our answers in the examples above satisfy the properties.
Illustrative Example 1: The inverse of f(x) = 5x – 8 is ( )
Let us check our answer using composite function ( ( )).
Solutions: f(x) = 5x – 8 and ( ) Given
( ( )) Composite Function
. / ( ) Replace ( )

. / ( ) Simplify the equation


. / Simplify the equation
Since our final answer is x, we therefore conclude that the inverse of f(x) = 5x – 8 is
( ) and vice versa.

Page 43
To develop more your skills in determining the inverse of a function, please try this
challenge.

DIRECTIONS: Determine the inverse of the following functions. Copy ALL and write the
answers on your gen Math notebook. (5 points each)

1. ( )
Points Rubrics
2. ( ) 1 point Correct step 1
3. ( ) 1 point Correct step 2
2 points Correct step 3
4. ( ) 1 point Incorrect step 3
1 point Correct step 4

Score Description
Now, check your work by turning to page 50 for the 18-20 Naol perfect!
key to correction. How many correct answers did you get? 14-17 Bongga ka!
Rate your result using the table above. If your score is at 8-13 Pwede na besh
least 10 out of 20, you may now proceed to next part of the Practice pa more
discussion. 0-7
on page 49 sec J

E. Discussing new concepts and practicing new skills #2

WEEK 4: Day 3

Lesson 13: Graphs of Inverse Function


As you observe in the previous examples, we need to determine the shape of an
equation because we all know that a function has inverse if and only if it is one-to-one function.
Let us study more about the graph of inverse function.
In graphing an equation, we need to use table of values as being discuss below.
Illustrative Example 1: Graph the function ( ) and its inverse.
In this case, first, let us complete the table of values below.
x -3 -2 -1 0 1 2 3
y

To complete the table of values, evaluate the equation using the values of x, as shown below.

If x = –3 If x = –2 If x = – 1 If x = 0
( ) ( ) ( ) ( )
( ) ( ) ( ) ( ) ( ) ( ) ( ) ( )
( ) ( ) ( ) ( )

Do you still remember now how to evaluate function? We can now complete the table of values.

x -3 -2 -1 0 1 2 3
y -5 -3 -1 1 3 5 7

If we will plot the points on the Cartesian plane, this will be the
shape of ( ) .

Given the graph of ( ) we can say that this


graph is a one-to-one function using a horizontal line test. In
addition, the graph of its inverse can be obtained by reflecting
the graph of .

Page 44
Solutions:

Take the reflections of the graph of across the line


of (the broken line)

Illustrative Example 2: Graph the function and its inverse.

Let us set values of x {x= -0.6, -0.4. -0.2, 0, 0.2, 0.4,…, 1.2} in order to make a table of
values. Study the table below.

X -0.6 -0.4 -0.2 0 0.2 0.4 0.6 0.8 1 1.2


Y -1.21 -1.06 -1.01 -1 -0.99 -0.94 -0.78 -0.49 0 -0.73

Using the table of values, we can plot them in the Cartesian plane as shown below.

Solution: Applying the horizontal line test, we confirm that this is a one-to-one function.
Now, let us reflect the graph across the line to get the plot of the inverse function.

To get the equation of the function


reflected, let us determine the inverse of the
function ( )

( )
Step 1
Step 2
Step 3

( ) √ Step 4

The inverse of the function


( ) is ( ) √

REMEMBER ME!
To graph the inverse of the functions, the graph of the original function is just reflected
across the line y=x.

Page 45
WEEK 4: Day 4
Lesson 14: Solving Problems

Problem 1: Engineers have determined that the maximum force in tons that a particular bridge
can carry is related to the distance in meters between it supports by the following function:
( ) ( )
How far should the supports be if the bridge is to support 6.5 tons? Construct an inverse
function to determine the result.
Solution: ( ) . / Given

. / Step 1: Replace t(d) with t

To lessen confusion in this case, let us not interchange and as they denote
specific values. Solve instead for d in terms of t.

√ Take the cube root of both sides


√ Cross multiplication
Divide both side by √

( ) Step 4: Replace d as d(t)

Answer (a): The inverse function is ( )

Solution: Substitute t = 6.5
( ) Evaluation of function

( ) Express as decimal
Answer (b): The supports should be placed at most 6.70 meters apart.

Problem 2: The formula for converting Celsius to Fahrenheit is given as where C


is the temperature in Celsius and F is the temperature in Fahrenheit. Find the formula for
converting Fahrenheit to Celsius. If the temperature in a thermometer reads 109.4°F, what is
that in °C?
Solutions: Given
Add -32 both sides
(Addition Property of Equality)
( ) Multiply 5/9 both sides
(Multiplication Property of Equality)
To determine the temperature in °C, we need to evaluate the function.
( ) Substitute the value of F
( ) Subtraction
Multiplication

Answer: The temperature 109.4°F is equal to 43°C

F. Developing mastery
DIRECTIONS: Write TRUE if the statement is correct; otherwise, write FALSE on the blank
provided before each number. Copy ALL and write the answers on your Gen Math notebook.
(2 points each)
______________1. The relation pairing a person to his or her citizenship is a function.
______________2. The relation of books to authors is an example of one-to-one function.
______________3. The inverse of the set {(0,4), (1,5), (2,6),…, 6,10)} is a one-to-one function.
______________4. The inverse of ( ) is ( )
______________5. The inverse of the rational function ( ) is ( ) .
______________6. A function has an inverse if and only if it is one-to-one function.
______________7. In order to determine if the graph is one-to-one function, vertical line test
should be used.

Page 46
______________8. The graph of the inverse of a function can be obtained by reflecting the
graph of the function across .
______________9. The inverse of the function ( ) ( ) is ( ) √ .
______________10. The composite of the function and its inverse is always equal to x.

Score Description Now, check your work by turning to page 51 for the
20 Naol perfect! key to correction. How many correct answers did you get?
12-19 Bongga ka!
8-11 Pwede na besh
1- 7 Practice pa more
G. Finding practical applications of concepts and skills in daily living

Did you know that Inverse function is interrelated to other discipline?


Here are some examples of topics.

 In English, our teachers taught us antonyms where a word opposite in meaning


to another (e.g., beautiful and ugly, good and bad)
 Conversion of measurements (e.g., Fahrenheit and Celsius, Liters to gallons, and
others)
 Currency Exchanged

H. Making generalizations and abstractions about the lesson

KEY POINTS
 A one-to-one function is a function in which every element in the range of a function
corresponds to exactly one element in the domain.
 A simple way to determine if a given graph is that of a function is by using the Vertical
Line Test. On the other hand, use Horizontal Line Test to test that a function is a one-to-
one function.
 An inverse function is a function that “reverses” another function
 A function has an inverse if and only if it is one-to-one function.
 Properties of an inverse of a one-to-one function
Given a one-to-one function ( ) and its inverse ( ), then the following are true:
 The inverse of ( ) is ( )
 ( ( )) for all x in the domain of
 ( ( )) for all x in the domain of f

I. Evaluating learning WEEK 4: Day 5

GENERAL DIRECTIONS: This part is recorded and graded. Copy ALL the given using this
format on a separate yellow sheet of paper. The yellow paper will be submitted to your Math
teachers on the day of the scheduled retrieval.

ACTIVITY 4.1 IN GENERAL MATHEMATICS


(WEEK 4)
Name: _________________Section: _________ Parent‟s Signature: ___________________
COORDINATES
DIRECTIONS: Find the inverse of the following set of ordered pairs. On the blank before each
number, indicate if the resulting inverse is a function or just a mere relation.
_________________________1. A= {(-2,8),(0,5),(-5,1),(6,9),(3,7)}
_________________________2. B= {(-1,-7),(0,-6),(4,2),(5,3)}
_________________________3. C= {(1,5),(2,2),(3,-1),(1,-5)}
_________________________4. D= {(-4,-7),(0,5),(1,3),(2,5)}
_________________________5. E= {(1,0),(2,5),(2,-5),(3,7)}
_________________________6. F= {(-1,-1),(-3,-2),(-3,-3),(-4,-4),(-5,-5)}
_________________________7. G= {(1,-1),(3,-2),(4,-4),(7,-7),(10,-10)}
_________________________8. H= {(1,1),(0,-1),(1,1/2)}
_________________________9. I= {(-1,0),(9,1),(4,2),(1,0)}
_________________________10. J= {(-1,1),(-10,-1),(2,4),(0,0)}

Page 47
ACTIVITY 4.2 IN GENERAL MATHEMATICS
(WEEK 4)
Name: _________________Section: _________ Parent‟s Signature: ___________________
ALGEBRAIC INVERSES
DIRECTIONS: Find the inverse of each function. Show your complete solution.
11 – 12. f(x) = 14 - x
13 – 14. f(x) = 7x
15 – 16. f(x) =

17 – 18. f(x) =
19 – 20. 4x – 9y = 6

ACTIVITY 4.3 IN GENERAL MATHEMATICS


(WEEK 4)
Name: _________________Section: _________ Parent‟s Signature: ___________________
PROVING INVERSE FUNCTIONS
DIRECTIONS: Write “AWA” if the functions f and g are inverses and “ALI” if the two functions
are not inverses. Show your solutions.

_________21. f(x) = 12x g(x) = 12/x


_________22. f(x) = x – 5 g(x ) = 5 + x
_________23. f(x) = 5x – 8 g(x) =

_________24. f(x) = 5x - 1 g (x) =

_________25. f(x) = 5x – 3 g (x) =

_________26. f(x) = x + 12 g (x ) = 12x


_________27. f(x) = 5x g (x) =

_________28. f(x) = g (x ) = ∛x
_________29. f(x) = 3 + x g (x ) = 3 – x
_________30. f(x) =√ g(x ) = √

J. Additional activities for application or remediation

GENERAL DIRECTIONS: Copy ALL and write the answers on your Gen Math notebook.

Remedial Activity 4.1: ONE-TO-ONE FUNCTIONS


DIRECTIONS: Determine if the following relation represents ONE-TO-ONE functions or not.
Write your answers on the blank provided before each number.
__________ 1. M = {(7, -5), (6, -3), (5, -1), (4, 1)}
__________ 2. A = {(6, 0), (6, 1), (6, 2), (6, 3), …}
__________ 3. The relation pairing a PHS student to his or her class section.
__________ 4. The relation pairing a company to its official logo.
__________ 5. The relation pairing a counting number to its opposite.
__________ 6. X Y
h
10
5 12
14
3 16
18

Page 48
Remedial Activity 4.2: ONE-TO-ONE FUNCTIONS IN REAL-LIFE
DIRECTIONS: Put a check mark on the blank provided before each number if each relation
represents a one-to-one function.

__________ 1. Person to Citizenship


__________ 2. Family to their residence
__________3. The width of a rectangle is five more than its length. Let be the function mapping
the length of the rectangle to its area.
__________ 4. Consider the uppercase letter of English Alphabet W as a graph.
__________ 5. y = 3x2 + 8x -9

Remedial Activity 4.3: ALGEBRAIC INVERSES


DIRECTIONS: Determine the inverse of the following functions. Write your complete solutions.

1. ( )

2. ( )

3. ( )

4. ( )

APPENDIX A
KEY TO CORRECTION

CHALLENGE 4.1:

NOT 1. M = {(3, -1), (2, -1), (1, -1), (0, -1)}


ONE TO ONE 2. A = {(0, 0), (2, 1), (4, 2), (6, 3), …}
ONE TO ONE 3. The relation pairing a PHS student to his or her student number.
ONE TO ONE 4. The relation pairing an airport to its airport code
NOT 5. The relation pairing a real number to its square.

NOT 6. X Y
h
11
7 13
17
2 19
23

CHALLENGE 4.2:

 1). True or false questions to answers


NOT 2). While reading your textbook, you find a function that has two inputs that
produce the same answer.
 3). The length of a rectangle is four more than its width. Let be the function
mapping the length of the rectangle to its area.
NOT 4). Consider the uppercase letter of English Alphabet S as a graph.
NOT 5). y = x2 + 4x -3

Page 49
CHALLENGE 4.3:

1. ( )
Step 1 (1 point)
Step 2 (1 point)
Step 3 (2 points)

( ) Step 4 (1 point)

2. ( )

( ) √

3. ( )

( )

( )

( )

4. ( )

( )

( )

Page 50
APPENDIX B
KEY TO CORRECTION
F. DEVELOPING MASTERY

FALSE1. The relation pairing a person to his or her citizenship is a function.


The relation is not a function because a person can have dual citizenship.
FALSE 2. The relation of books to authors is an example of one-to-one function.
The relation is not a one-to-one function because books can have multiple
authors that wrote the book.
TRUE 3. The inverse of the set {(0,4), (1,5), (2,6),…, 6,10)} is a one-to-one function.
The inverse of the set is {(4,0), (5,1), (6,2),…, 10,6)}. This is one-to-one because
every element in the range of a function corresponds to exactly one element in
the domain.
FALSE. 4. The inverse of ( ) is ( )
Solutions: ( )



( ) √
Therefore, the inverse of ( ) is ( ) √ .
TRUE 5. The inverse of the rational function ( ) is ( ) .
Solutions: ( )

( )

( )

( )
TRUE 6. A function has an inverse if and only if it is one-to-one function.
FALSE 7. In order to determine if the graph is one-to-one function, vertical line test
should be used.
Vertical Line Test – A relation is a function if each vertical line does not intersect
the graph at more than one point.
Horizontal Line Test – A function is a one-to-one if each horizontal line does not
intersect the graph at more than one point.
TRUE 8. The graph of the inverse of a function can be obtained by reflecting the graph of
the function across .
FALSE 9. The inverse of the function ( ) ( ) is ( ) √ .
Solutions: ( ( )) √ .
(( ) ) √( )
(( ) ) ( )
(( ) )
Since it doesn‟t satisfy the property of inverse function (read item 10), then The
inverse of the function ( ) ( ) is not ( ) √ .
TRUE 10. The composite of the function and its inverse is always equal to x.

Page 51
SELF-INSTRUCTIONAL PACKETS
GENERAL MATHEMATICS GRADE 11

School Teaching Dates/ Week


September 21 – 25, 2020
PAMPANGA HIGH SCHOOL (Week 5)
Teacher General Mathematics Teachers Quarter First

I. OBJECTIVES

A. Content Standards
The learner demonstrates understanding of key concepts of exponential functions.

B. Performance Standards
The learner is able to apply the concepts of exponential functions to formulate and
solve real-life problems with precision and accuracy.

C. Learning Competencies

The learner
1. represents real-life situations using exponential functions; (M11GM-Ie-3)
2. distinguishes between exponential function, exponential equation, and
exponential inequality; (M11GM-Ie-4)
3. solves exponential functions and inequalities; (M11GM-Ie-f-1) and
4. represents an exponential function through its table of (a) values, (b) graph, and
(c) equation. (M11GM-If-2)

D. Objectives

At the end of the lesson, the learners should be able to:


1. represent real-life situations using exponential functions;
2. distinguish between exponential function, exponential equation, and exponential
inequality;
3. solve exponential functions and inequalities; and
4. represent an exponential function through its (a) table of values, (b) graph, and
(c) equation.

II. CONTENT
EXPONENTIAL FUNCTIONS

Learning Resources

A. Reference
Learner‟s Material in General Mathematics, pp. 77-98
Teacher‟s Guide in General Mathematics, pp. 87-111

B. Other Learning Resources


Conceptual Math & Beyond General Mathematics, pp. 76-92

III. PROCEDURES
WEEK 5: Day 1

A. Reviewing previous lesson or presenting the new lesson

There are many real-life situations that can be modelled using functions. In your previous
lessons about functions, you have learned how to model these situations using linear functions,
quadratic functions, polynomial functions, and rational functions. In this lesson, you shall learn
another type of function which is the exponential function.

To jog your memory, let us start by recalling the laws of exponents.

Page 52
1) Product of powers: To multiply when two bases are the same, you write the base and add
the exponents.

am • an = am + n

Examples

A. x3 • x8 = x3 + 8 = x11 B. 24 • 22 = 24 + 2 = 26 C. x2y • x3y4 = (x2 + 3)(y1 + 4) = x5y5

2) Quotient of powers: To divide when two bases are the same, you write the base and
subtract the exponents.

= am – n

Examples

A. = x 5 – 2 = x3 B. = 35 – 3 = 32 C. = (x2 – 1)(y5 – 3) = xy2

3) Power of a power: To raise a power to another power, you write the base and multiply the
exponents.

(am)n = amn

Examples

A. (x3)2 = x(3)(2) = x6 B. (32)4 = 3(2)(4) = 38 C. (z5)2 = z(5)(2) = z10

4) Power of a product: When the base is a product of two or more variables, you raise each
variable by the given exponent.

(ab)m = ambm

Examples

A. (2a)3 = (2)3(a)3 = 8a3 B. (6x3)2 = (6)2(x3)2 = 36x6

5) Power of a quotient: It is equal to the quotient obtained when the dividend and divisor are
each raised to the indicated power separately, before the division is performed.

. /

Examples

( ) ( ) ( ) ( )
A. . / B. . / ( )
( ) ( ) ( )

6) Zero exponent rule: Any base (except zero) raised to the zero power is equal to one.

a0 = 1, a ≠ 0

Examples:

A. y0 = 1 B. 60 = 1 C. . /

7) Negative exponent rule: To change a negative exponent to a positive one, flip it into a
reciprocal.

a-n = ,a≠0

Page 53
Examples:

A. x-4 = = B. 2x-1 = ( ) . / ( ). /

( )
C. ( )
( ) ( ) ( )

Now it‟s your time to do your task! Try to answer the challenge. Good luck!

DIRECTIONS: Simplify the following expressions. Copy ALL and write write the answers on your
Gen Math notebook. (1 point each)
1) x5 • x4 6) (2y4)3

2) y • y5 7) . /

3) 8) . /

4) 9) 4x-3

5) (x5)2 10) (2x2)-2


Score Description
Now, check your work by turning to page 64 for 10 Naol perfect!
the key to correction. How many correct answers did you 8-9 Bongga ka!
get? Rate your result using the table on the right. If your
5-7 Pwede na besh
score is at least 5 out of 10, you may now proceed to next
part of the discussion. Practice pa more
0-4
on page 63 sec J

B. Establishing a purpose for the lesson


In 1859, 24 rabbits were imported to Australia from Europe as a new source
of food. Rabbits are not native to Australia, but conditions there were ideal
for rabbits and so they flourished. Soon, there were so many rabbits that they
damaged grazing land. By 1887, the government was offering a reward for a way to control the
rabbit population. How many rabbits might there have been in 1887?

This problem is an example of exponential growth and it is one of the applications of


exponential function.

C. Presenting examples/instances of the new lesson


“Life itself is exponential.” - Jeff Rich
Do you agree with this quote? The spread of COVID-19 can actually be modelled thru
exponential functions. In this part of the lesson, we will learn how to apply exponential functions
in real-life.
Lesson 15: Representing real-life situations using exponential functions.

An exponential function is a function in which the independent variable is an exponent.


It is in the form f(x) = bx or y = bx where b > 0 and b ≠ 1.

The rabbit population example is said to grow exponentially. In exponential growth, the
original amount is repeatedly multiplied by a positive number called the growth factor. Let us
discuss how you can construct an exponential model.

Page 54
Exponential Models and Population Growth

Problem 1. Going back to the opening problem, let us assume that the rabbit population
doubles every year. a) Give an exponential model for the number of rabbits per year. b) How
many rabbits would there be after 28 years (from 1859 to 1887)?

Initially there were 24 rabbits. So at 0 year, the number of rabbits is 24. Since the rabbit
population doubles every year, that means after the first year there will be (24)(2) = 48 rabbits.
On the second year there will be (48)(2) or we can also write it as (24)(2)(2) = 96 rabbits. Study
the table below and observe if you can see a pattern in every solution.

Initially, at t = 0 Number of rabbits 24


=
at t = 1 year Number of rabbits 24(2) = 48
=
at t = 2 years Number of rabbits 24(2)(2) = 24(2)2 = 96
=
at t = 3 years Number of rabbits 24(2)2(2) = 24(2)3 = 192
=
at t = 4 years Number of rabbits 24(2)3(2) = 24(2)4 = 384
=
at t = 5 years Number of rabbits 24(2)4(2) = 24(2)5 = 768
=
at t = 6 years Number of rabbits 24(2)5(2) = 24(2)6 = 1536
=
⁞ ⁞ ⁞
at t = 28 years Number of rabbits 24(2)27(2) = 24(2)28 = 6,442,450,944
=

Did you notice something in the table? Yes, the initial number of rabbits (24) is always
present in every computation. What else? Did you notice the number of years and the
exponents in the solution are the same? The number of years becomes the exponents of 2. But
where does the number 2 came from? That is right, it came from the condition that every year
the number of rabbits double.

Now that we have all the variables in the problem, let us now form the exponential model
for this problem.

a) The exponential function is in the form f(x) = bx. In the problem, f(x) represents the number of
rabbits, while b is the number 2 and x is the number of years. Let us use the solution on the
second year.

96 = 24(2)2 Since 96 is f(x) and 2 is b then,


f(x) = 24(b)x= 24(2)x The exponential model for this situation is f(x) = 24(2) x or y = 24(2)x.

b) So if x is 28 years, the number of rabbits will be, y = 24(2) 28 = 6, 442,450,944

TAKE NOTE: Suppose a quantity f(x) or y doubles every T units of time. If yo is the initial
amount, then the quantity y after x units of time is given by the exponential model, ( ) .

Problem 2. What if the number of rabbits in the problem triple every year, how are you going to
represent the exponential model?

For this problem, yo is 24, T is 1 year, while the base will be 3 (triple) instead of 2.
Therefore, ( ) and the model will be ( ) ( ) .

Exponential Functions and Half-Life

Another application of exponential function is the half life of a substance. This is the time it takes
for a substance to exponentially decay to half of its original quantity. We use half-life in
applications involving radioactive isotopes.

Page 55
TAKE NOTE: If the half-life of a substance is T units, and y o is the initial amount of the
substance when x = 0, then the amount of y of substance remaining after x units of time is given
by the exponential model, ( ) .

Problem 3. Suppose that the half-life of a certain radioactive substance is 10 days and there
are 10 g initially, a) give an exponential model for the amount of remaining substance and b)
determine the amount of the substance remaining after 30 days.

a) In this problem, 10 g is the initial amount y o, while 10 days is the value of T and x is 30 days.
So using the exponential model ( ) , the answer will be ( ) .

b) If x is 30, then after 30 days the amount of the substance will be ( ) ( )


( ) g.

Compound Interest

Benjamin Franklin, described compound interest: "Money makes money. And the money
that money makes, makes money." That is probably the simplest explanation of compound
interest you'll ever hear. You will encounter more about compound interest on the second
quarter, but for now let us consider its exponential model.

TAKE NOTE: If a principal P (amount deposited or loaned) is invested at an annual rate of r,


compounded annually, then the amount f(x) after x years is given by the exponential model
( ) ( ) .

Problem 4. Monkey D. Luffy invested ₱100,000 in a company that offers 6% interest


compounded annually. a) Define an exponential model for this situation. b) How much will this
investment be worth after five years?

a) In this problem, the ₱100,000 amount invested is P, while r is 6% or 0.06. Using the
exponential model ( ) ( ) , for this condition will have ( ) ( ) .
The final answer will be ( ) ( ) .

b) After 5 years (when x = 5), the amount of the investment will be,
( ) ( ) or ₱133,822.56.

DIRECTIONS: Writing exponential functions. Solve the following scenarios. Copy ALL and
write the answers on your Gen Math notebook.(2 points each)

1) Alexander Fleming discovered penicillin by observing mold growing on Petri dishes. Imagine
that you are biochemist studying a type of mold that has 3,000 spores initially. Suppose that
the mold spores quadruple (increases 4 times) every 20 minutes. a) Give an exponential model
for the amount of spores as a function of time. b) How many mold spores would there be after
1 hour (60 minutes)?
2) A round goby is a bottom-dwelling fish native to Eastern Europe. In 1995, it was found in
the Great Lakes, where it is expected to be harmful to already existing habitats. The round
goby is known to spawn several times during the summer, and biologists are tracking the
growth of its population. Suppose 11 round gobies were in the Great Lakes in 1995 and that
their population triples in size each year. a) Write an exponential growth equation to describe
this situation. b) How many round gobies were in the Great Lakes after 2 years?
3) Cesium 137 is a radioactive metal with a short half-life of 30 years. In a sample originally
having 10 g of cesium 137, a) give an exponential model for the amount of remaining
substance and b) how much cesium 137 will be present after 120 years?

Page 56
Score Description
Now, check your work by turning to page 64 for the key 6 Naol perfect!
to correction. How many correct answers did you get? Rate 4-5 Bongga ka!
your result using the table on the right. If your score is at least 3
2-3 Pwede na besh
out of 6, you may now proceed to next part of the discussion.
Practice pa more
0-1
on page 63 sec J
In the next part of the lesson, you will be learning the difference between exponential
function, equations and inequalities.
WEEK 5: Day 2

Lesson 16: Exponential Functions, Equations and Inequalities

An exponential function is a function in which the independent variable is an exponent.


An exponential equation is an equation in which a variable occurs in an exponent. An
exponential inequality is an inequality in which a variable occurs in an exponent.

Illustrative Example 1: In the rabbit population problem in the beginning of the lesson, the
exponential function is f(x) = 24(2)x or y = 24(2)x. This will become an exponential equation if we
change f(x) to a particular value, for example 96. The exponential equation will be 96 = 24(2) x.
For the exponential inequality, the symbols <, >, ≤, and ≥ are used instead of the = sign. So the
exponential inequality will be 96 < 24(2)x. Observe the table below to see the difference.

Exponential function Exponential equation Exponential inequality

96 < 24(2)x
96 > 24(2)x
f(x) = 24(2)x or y = 24(2)x 96 = 24(2)x 96 ≤ 24(2)x
96 ≥ 24(2)x

Other Examples:
1) 5x > 25x + 1 Exponential inequality

2) y = 162x + 3 Exponential function

3) 125 = 25x + 1 Exponential equation

4) f(x) = 2x3 None of these

5) 27x ≤ 3 x + 1 Exponential inequality

6) 10x = 1002x + 1 Exponential equation

DIRECTIONS: Determine whether the given is an exponential function, an exponential


equation, an exponential inequality, or none of these. Copy ALL and write the answers on your
Gen Math notebook. (1 point each)
1. f(x) = 5x2
2. 2 ≥ (1/2)x Score Description
6 Naol perfect!
3. 74x = y
4-5 Bongga ka!
4 4(10x) = 500 2-3 Pwede na besh
x+3
5 7 < 14 Practice pa more
0-1
6 y = 0.5x on page 63 sec J

Page 57
Now, check your work by turning to page 64 for the key to correction. How many correct
answers did you get? Rate your result using the table on the previous page. If your score is at
least 3 out of 6, you may now proceed to next part of the discussion.

D. Discussing new concepts and practicing new skills #1

WEEK 5: Day 3

Lesson 17: Solving Exponential Equations and Inequalities

17.1: Solving Exponential Equations

To solve exponential equation, we use the laws of exponents. We are also going to use
this property that if x and y are real numbers and if b x = by, then we can say that x = y.

Illustrative Example 1. Let us solve the equation 64 = 8 x – 2.

Rewrite each side of the equation as an exponential


Step 64 = 8x – 2
expression having the same base. 64 is the same as
1 82 = 8x – 2
82.
If 82 = 8x – 2, then Use the law of exponents. If bx = by, then we can say
2 = x – 2 (Add both sides by 2) that x = y. In bx = by, b is 8 in this example. To get the
Step 2+2=x–2+2 equation x = y just cover the number 8 and the
2 4=x exponents on each side will be the equation x = y.
Thus x = 4 is the solution of 82 = 8x – 2 and that‟s how you get 2 = x – 2.
the equation 64 = 8x – 2 Solve the value of x.
64 = 8x – 2
Step 64 = 84 – 2
Check the solution by substituting x to the equation.
3 64 = 82
64 = 64
Illustrative Example 2. Let us determine the value of x in the equation 25 2x = 125.
Rewrite each side of the equation as an exponential
252x = 125 expression having the same base. The exponential
Step (52)2x = 53 (Use power of a power form of 25 is 52, but 5 is not equal to 125. Try to
1 to simplify) express 125 in exponential form. You should get 5 3. So
54x = 53 if we change 25 to 52 and 125 to 53, we will now have
the same base.
If 54x = 53, then Use the law of exponents. If bx = by, then we can say
4x = 3 (Divide both side by 4) that x = y. In bx = by, b is 5 in this example. To get the
Step equation x = y just cover the number 5 and the
2 exponents on each side will be the equation x = y.
Thus x = is the solution of 4x 3
5 = 5 and that‟s how you get 4x = 3.
the equation 252x = 125 Solve the value of x.
252x = 125
. /
Step
Check the solution by substituting x to the equation.
3 . /

125 = 125
Illustrative Example 3. Let us answer the equation (½)x+2 = 16x.

(½)x+2 = 16x Rewrite each side of the equation as an exponential


expression having the same base. The exponential
(½)x+2 = (24)x (Use negative form of 16 is 42, but 4 is not equal to ½. 16 is also the
Step (2-1)x+2 = 24x exponent rule same as 24, but 2 is still not equal to ½. Let us apply
1 -x -2 4x the negative exponent rule on (½)x+2, what will you
(2) =2 and power of a
get? ½ will become 2-1 and (½)x+2 is the same as
power to simplify) (2-1)x+2. So if we change (½)x+2 to (2-1)x+2 and 16x to
(24)x, the bases on each side will be the same.

Page 58
If (2)-x - 2 = 24x, then
-x - 2 = 4x (Add both side by x)
Use the law of exponents. If bx = by, then we can say
-x + x - 2 = 4x + x that x = y. In bx = by, b is 2 in this example. To get the
Step -2 = 5x (Divide both side by 5)
equation x = y just cover the number 2 and the
2 exponents on each side will be the equation x = y.
2-x - 2 = 24x and that‟s how you get -x - 2 = 4x.
Thus x = is the solution of
Solve the value of x.
the equation (½)x+2 = 16x
(½)x+2 = 16x

( )

( )

Step
Check the solution by substituting x to the equation.
3 ( ) ( )

( )

( )

Now it‟s your time to do your task! Try to answer the next challenge. Good luck!

DIRECTIONS: Solve the following equations. Copy ALL and write the answers on your Gen
Math notebook. (2 points each)

1. 32x = 81 Score Description


2. 2 x+1
= 64 8 Naol perfect!
5-7 Bongga ka!
3. 3x =
1-3 Pwede na besh
4. 42x = Practice pa more
0
on page 63 sec J

Now, check your work by turning to page 64 for the key to correction. How many correct
answers did you get? Rate your result using the table above. If your score is at least 4 out of 8,
you may now proceed to next part of the discussion.

WEEK 5: Day 4

E. Discussing new concepts and practicing new skills #2


17.2 Solving exponential inequalities

To solve an exponential inequality, we use the following properties:


a. If b > 1 and bx < by, then x < y.
b. If b > 1 and bx > by, then x > y.
Notice that the properties for inequalities also hold when < is replaced by ≤ and > is replaced
by ≥.

Page 59
Illustrative Example 1. Let us determine the value of x in the inequality 3 x > 32x – 3.
In this example, b is equal to 3 and 3 > 1 so we use either property a or b. The inequality sign
used is > which makes as finally apply the second property (b. If b > 1 and b x > by, then x > y).

Use property b where


b > 1 and bx > by, then
3x > 32x – 3 x > y.
Step 1
x > 2x – 3 (Using property b) bx is 3x while by is 32x – 3
which will make x > y
same as x > 2x – 3.
x > 2x – 3
x – 2x > 2x – 2x – 3 (Add –2x on both sides)
–x > –3 (Divide both sides by -1) Solve for the value of x
Step 2
(When you divide by a negative number, the sign of the inequality must flip) in the inequality.
x<3
Therefore the solution for 3x > 32x – 3 is x < 3.

Illustrative Example 2. Let us solve the value of x in the inequality 82x – 1 ≤ 16x.
In this example, we need to rewrite each side of the equation as an exponential expression
having the same base. The exponential form of 8 is 23, while 16 is also the same as 24. That will
make 82x – 1 ≤ 16x equal to (23)2x – 1 ≤ (24)x. We simplify by applying the power of a power rule and
we will have 26x – 3 ≤ 24x. b is equal to 2 and 2 > 1 so we use either property a or b. The
inequality sign used is ≤ which makes as finally apply the first property (a. If b > 1 and bx < by,
then x < y).

Use property a where


b > 1 and bx ≤ by, then
82x – 1 ≤ 16x
x ≤ y.
Step 1 26x – 3 ≤ 24x
bx is 26x – 3 while by is
(Using property a)
6x – 3 ≤ 4x
24x which will make x ≤
y same as 6x – 3 ≤ 4x.
6x – 3 ≤ 4x (Add –4x + 3 on both sides)
6x – 3 – 4x + 3 ≤ 4x – 4x + 3
2x ≤ 3 (Divide both sides by 2)

Solve for the value of


Step 2
x in the inequality.

Therefore the solution for 82x – 1 ≤ 16x is .

DIRECTIONS: Solve the following inequalities. Copy ALL and write the answers on your Gen
Math notebook (2 points each)
Score Description
1. 3x < 9x – 2 6 Naol perfect!
x x+1
2. 5 > 25 4-5 Bongga ka!
2-3 Pwede na besh
3. . / Practice pa more
0
on page 63 sec J

Now, check your work by turning to page 66 for the key to correction. How many correct
answers did you get? Rate your result using the table above. If your score is at least 3 out of 6,
you may now proceed to next part of the discussion.

Page 60
F. Developing mastery

GENERAL DIRECTIONS: Copy ALL and write the answers on your Gen Math Notebook.

DIRECTIONS: Writing exponential functions. Solve the following scenarios. (2 points each)
1-2. Nobelium, an element discovered in 1958, has a half-life of 10 minutes under certain
conditions. In a sample containing 5 g of nobelium, a) give an exponential model for the amount
of remaining substance and b) how much nobelium is left after 40 minutes?

3-4. On Elizabeth‟s sixth birthday, her grandparents present her with a ₱3,000 certificate of
deposit (CD) that earns 5.09% interest, compounded annually. a) Define an exponential model
for this situation. b) If the CD matures on her sixteenth birthday, what amount will be available
then?

5-6. The price of a Honda Civic is around ₱1,115,000. Assuming that the value of the car each
year is 90% of the value of the preceding year, a) define an exponential model for the value of
the car after x years and b) what will be the value of the car after 10 years?

DIRECTIONS: Solve the following exponential equations. (3 points each)


7-9. 3x + 1 = 81

10-12. 42x – 1 = 8x + 3

13-15. 57 – x = 125

DIRECTIONS: Solve the following exponential inequalities. (2 points each)


16-17. 43x + 2 < 64
Score Description
x
18-19. 10 > 100 -2x – 5 21 Naol perfect!
13-20 Bongga ka!
20-21. 9x ≤ 92x – 1 8-12 Pwede na besh
1- 7 Practice pa more
Now, check your work by turning to page 67 for the key to correction. How many correct
answers did you get?

G. Finding practical applications of concepts and skills in daily living

Why exponential growth makes the perfect case for social distancing

Understanding exponential growth illustrates why social distancing is vitally important


during the covid-19 pandemic. If we practice social distancing, where we avoid large gatherings
and keep a significant distance between ourselves and others' bodies, we can reduce the
transmission rate. Interventions such as this can effectively decrease the "doubling time" of the
disease significantly, slowing the rate of infection in the uninfected population. The more
successful an intervention such as this is, the more spread out in time the infection will be
among the public: what infectious disease specialists are calling "flattening the curve," which
prevents everyone from getting sick at once. This didn't happen in countries like Italy and Iran
during a critical period, and explains why both infection rates and death rates are so high in
those countries.

H. Making generalizations and abstractions about the lesson

 An exponential function is a function in which the independent variable is an exponent.

 An exponential equation is an equation in which a variable occurs in an exponent.

 An exponential inequality is an inequality in which a variable occurs in an exponent.

Page 61
WEEK 5: Day 5
I. Evaluating learning

GENERAL DIRECTIONS: This part is recorded and graded. Copy ALL the given using this
format on a separate yellow sheet of paper. The yellow paper will be submitted to your Math
teacher on the day of the scheduled retrieval.

ACTIVITY 5.1 IN GENERAL MATHEMATICS


(WEEK 5)
Name: _________________Section: _________ Parent‟s Signature: ___________________
REPRESENTING REAL-LIFE SITUATIONS USING EXPONENTIAL FUNCTIONS

DIRECTIONS: Write the following situations as exponential functions and solve the problems.

1. Originally, a sample has 10g of radium with a half-life of 1620 years. a) Give an exponential
model for the amount of remaining substance and b) how much radium will be present after
2430 years?

2. A barangay has 1,000 individuals and its population doubles every 60 years. a) Give an
exponential model for the barangay‟s population. b) What is the barangay‟s population in 10
years?

3. A bank offers a 2% annual interest rate, compounded annually, for a certain fund. a) Give an
exponential model for a sum of ₱10,000 invested under this scheme. b) How much money will
there be in the account after 20 years?

4. The half-life of a radioactive substance is 1200 years. If the initial amount of the substance is
300 grams, a) give an exponential model for the amount remaining after t years. b) What is the
remaining amount of substance after 1000 years?

5. Most cars depreciate as they get older. Suppose a certain car originally costs ₱1,990,000 and
its value decreases by 80% each year, a) define an exponential model for the value of the car
after x years and b) what will be the value of the car after 7 years?

ACTIVITY 5.2 IN GENERAL MATHEMATICS


(WEEK 5)
Name: _________________Section: _________ Parent‟s Signature: ___________________
SOLVING EXPONENTIAL EQUATIONS
DIRECTIONS: Solve the following equations.
1. 49x = 73x +1
2. 125x +1 = 25x
3. 9x – 1 = 27x
4. 22x + 1 = 16x
5. 100x = 1000x – 1

ACTIVITY 5.3 IN GENERAL MATHEMATICS


(WEEK 5)
Name: _________________Section: _________ Parent‟s Signature: ___________________
SOLVING EXPONENTIAL INEQUALITIES
DIRECTIONS: Solve the following inequalities.
1. 43x + 2 < 64
2. 23x + 4 > 22x
3. 82x – 3 ≥ 44x
4. 25x – 9 > 4x
5. 22x + 3 > 23x

Page 62
J. Additional activities for application or remediation
GENERAL DIRECTIONS: Copy ALL and write the answers on your Gen Math notebook.

Remedial Activity 5.1: LAWS OF EXPONENTS


DIRECTIONS: Simplify the following expressions.

1. x2 • x6 6. (3y4)2
2. y3 • y 7. . /

3. 8. . /

4. 9. 2x-2
5. (x2)5 10. (2x)-3

Remedial Activity 5.2: EXPONENTIAL FUNCTIONS IN REAL-LIFE


DIRECTIONS: Write the following situations as exponential functions and solve the problems.

1. A barangay has 1,500 individuals and its population doubles every 50 years. a) Give an
exponential model for the barangay‟s population. b) What is the barangay‟s population in 10
years?

2. A bank offers a 4% annual interest rate, compounded annually, for a certain fund. a) Give an
exponential model for a sum of ₱30,000 invested under this scheme. b) How much money will
there be in the account after 10 years?

3. The half-life of a radioactive substance is 1000 years. If the initial amount of the substance is
100 grams, a) give an exponential model for the amount remaining after t years. b) What
amount of substance remains after 3000 years?

Remedial Activity 5.3: EXPONENTIAL EXPRESSIONS


DIRECTIONS: Determine whether the given is an exponential function, an exponential
equation, an exponential inequality, or none of these.

1. 82x = y 4. 2(10x) = 100


2. 4x ≤ (2)x 5. 7 < 49x + 3
3. f(x) = 5x2 6. y = 0.5x – 1

Remedial Activity 5.4: SOLVING EXPONENTIAL EQUATIONS


DIRECTIONS: Solve the following equations.

1. 49x = 74x – 1
x+1 2x
2. 9 = 27
3. 22x + 1 = 16x
2x +1 x
4. 125 = 25
x x–2
5. 10 = 1000

Remedial Activity 5.5: SOLVING EXPONENTIAL INEQUALITIES


DIRECTIONS: Solve the following inequalities.
x+2
1. 4 <8
2. 23x + 4 > 42x
3x 2x – 5
3. 8 ≥4
4. 27x – 8 ≤ 8x
5. 25x + 4 > 23x

Page 63
APPENDIX A
KEY TO CORRECTION

CHALLENGE 5.1:
1. x5 • x4 Ans. x9 6. (2y4)3 Ans. 8y12

2. y • y5 Ans. y6 7. . / Ans.

3. Ans. x4 8. . / Ans. 1

4. Ans. y3 9. 4x-3 Ans.

5. (x5)2 Ans. x10 10. (2x2)-2 Ans.

CHALLENGE 5.2:

1a. ( )

1b. When x = 60, ( ) ( ) ( ) = 192,000 mold spores

2a. ( ) ( )
2b. When x = 2, ( ) ( ) ( ) = 99 gobies

3a. ( ) ( )

3b. When x = 120, ( ) ( ) 0.625 g of cesium 137

CHALLENGE 5.3:

1. None of these 4. Exponential equation


2. Exponential inequality 5. Exponential inequality
3. Exponential function 6. Exponential function

CHALLENGE 5.4:

1. 32x = 81

32x = 81 The exponential form of 81 is 3 4 then we will


Step 1
32x = 34 now have the same base.
Use the law of exponents. If bx = by, then we
If 32x = 34, then can say that x = y.
2x = 4 (Divide both side by 2)
In bx = by, b is 3 in this problem. To get the
equation x = y just cover the number 3 and
Step 2
the exponents on each side will be the
Thus x = 2 is the solution of the equation x = y.
equation 32x = 81 32x = 34 and that‟s how you get 2x = 4.
Solve the value of x.
32x = 81
Check the solution by substituting x to the
Step 3 32(2) = 81
equation.
34 = 81

Page 64
2. 2x + 1 = 64
2x + 1 = 64 The exponential form of 64 is 2 6 then we will
Step 1
2x + 1 = 26 now have the same base.
Use the law of exponents. If bx = by, then we
If 2x + 1 = 26 , then can say that x = y.
x+1=6 (Add both side by -1) In bx = by, b is 2 in this problem. To get the
x+1–1=6–1 equation x = y just cover the number 2 and
Step 2
x=5 the exponents on each side will be the
Thus x = 5 is the solution of the equation x = y.
equation 2x + 1 = 64 2x+1 = 26 and that‟s how you get x + 1 = 6.
Solve the value of x.
2x + 1 = 64
25 + 1 = 64 Check the solution by substituting x to the
Step 3
26 = 64 equation.
64 = 64
x
3. 3 =
The exponential form of 9 is 3 2, but we still
need to apply the negative exponent rule to
Step 1
which will become 3-2 then we will now have
the same base.
Use the law of exponents. If bx = by, then we
can say that x = y.
If 3x = 3-2, then In bx = by, b is 3 in this problem. To get the
x = -2 equation x = y just cover the number 3 and
Step 2 Thus x = -2 is the solution of the the exponents on each side will be the
equation equation x = y.
3x = 3-2 and that‟s how you get x = -2.
Solve the value of x.
32x = 81
Check the solution by substituting x to the
Step 3 32(2) = 81
equation.
34 = 81
4. 42x =
The exponential form of 4 is 2 2, but we still
need to apply the negative exponent rule to
Step 1 ( ) (Simplify using power of a power) which will become 2-1 then we will now have
the same base.
Use the law of exponents. If bx = by, then we
If 24x = 2-1, then can say that x = y.
4x = -1 (Divide both sides by 4)
In bx = by, b is 2 in this problem. To get the
equation x = y just cover the number 2 and
Step 2
the exponents on each side will be the
Thus x = is the solution of the equation x = y.
equation 24x = 2-1 and that‟s how you get 4x = -1.
Solve the value of x.

. /

. /
Check the solution by substituting x to the
Step 3
. / equation.
( )

Page 65
CHALLENGE 5.5:
1. 3x < 9x – 2
Rewrite 9 to 32
Simplify using power of a power rule
b is equal to 3 and 3 > 1 so we use
3x < 9x – 2
either property a or b. The inequality
3x < (32)x – 2
Step 1 sign used is < which makes us use the
3x < 34x – 4 (Use power of a power rule)
first property (a. If b > 1 and b x < by,
x < 4x – 4
then x < y).
bx is 3 x while by is 34x – 4 which will make
x < y same as x < 4x – 4.
x < 4x – 4 (Add –4x on both sides)
x – 4x < 4x – 4x – 4
–3x < –4 (Divide both sides by -3)
(When you divide by a negative number, the
Step 2 sign of the inequality must flip) Solve for the value of x in the inequality.

Therefore the solution for 3x < 9x – 2 is

2. 5x > 25x + 1
Rewrite 25 to 52
Simplify using power of a power rule
b is equal to 5 and 5 > 1 so we use
5x > 25x + 1
either property a or b. The inequality
5x > (52)x + 1
Step 1 sign used is > which makes us use the
5x > 52x + 2 (Use power of a power rule)
second property (b. If b > 1 and bx > by,
x > 2x + 2
then x > y).
bx is 5 x while by is 52x + 2 which will make
x > y same as x > 2x + 2.
x > 2x + 2 (Add –2x on both sides)
x – 2x < 2x – 2x + 2
–x < 2 (Divide both sides by -1)

(When you divide by a negative


Step 2 Solve for the value of x in the inequality.
number, the sign of the inequality must flip)

Therefore the solution for 3x < 9x – 2 is


.
3. . /

Rewrite 25 to 52 and apply the negative


exponent rule to and it will become 5-1.
Simplify using power of a power rule
. / b is equal to 5 and 5 > 1 so we use
Step 1 ( ) ( ) either property a or b. The inequality
(Use power of a power rule) sign used is ≥ which makes us use the
–x – 3 ≥ –2x second property (b. If b > 1 and b x ≥ b y,
then x ≥ y).
bx is 5-x – 3 while by is 5–2x which will
make x ≥ y same as –x – 3 ≥ –2x.
–x – 3 ≥ –2x (Add 2x + 3 on both sides)
–x – 3 + 2x + 3 ≥ –2x + 2x + 3
–x ≥ 3 (Divide both sides by -1)

(When you divide by a negative number,


Step 2 the sign of the inequality must flip) Solve for the value of x in the inequality.

Therefore the solution for . /


is .

Page 66
APPENDIX B
KEY TO CORRECTION
F. DEVELOPING MASTERY

Writing exponential functions. Solve the following scenarios. (2 points each)

1. ( ) . /

2. When x is 40, ( ) . / . /
3. ( ) ( ) ( )
4. When x is 10, ( ) ( )
5. ( ) ( )
6. When x is 10, ( ) ( )

Solve the following exponential equations. (3 points each)

7-9. 3x + 1 = 81
3x + 1 = 81 The exponential form of 81 is 3 4 then we will
Step 1
3x + 1 = 34 now have the same base.
Use the law of exponents. If bx = by, then we
can say that x = y.
If 3x +1 = 34, then
In bx = by, b is 3 in this problem. To get the
x + 1 = 4 (Add both sides by -1)
equation x = y just cover the number 3 and
Step 2 x+1–1=4–1
the exponents on each side will be the
Thus x = 3 is the solution of the
equation x = y.
equation 3x + 1 = 81
3x + 1 = 34 and that‟s how you get x + 1 = 4.
Solve the value of x.
3x + 1 = 81
Check the solution by substituting x to the
Step 3 33 + 1 = 81
equation.
34 = 81

10-12. 42x – 1 = 8x + 3
42x – 1 = 8x + 3
2 2x – 1
= (23)x + 3 (Simplify using The exponential form of 4 is 2 2 and 8 is 23,
Step 1 (2 ) power of a power) then we will now have the same base.
24x – 2 = 23x + 9
Use the law of exponents. If bx = by, then we
can say that x = y.
If 24x – 2 = 23x + 9, then In bx = by, b is 2 in this problem. To get the
4x – 2 = 3x + 9 (Add both sides by -3x + 2) equation x = y just cover the number 2
Step 2 4x – 2 – 3x + 2 = 3x + 9 – 3x + 2 and the exponents on each side will be the
Thus x = 11 is the solution of the equation x = y.
equation 42x – 1 = 8 x + 3 24x – 2 = 23x + 9 and that‟s how you get 4x – 2 =
3x + 9.
Solve the value of x.
42x – 1 = 8x + 3
42(11) – 1 = 811 + 3
422 – 1 = 814 Check the solution by substituting x to the
Step 3
421 = 814 equation.
(22)21 = (23)14
242 = 242

Page 67
13-15. 57 – x = 125
57 – x = 125 The exponential form of 125 is 5 3 then we will
Step 1
57 – x = 53 now have the same base.
Use the law of exponents. If bx = by, then we
If 57 – x = 53, then can say that x = y.
7 – x = 3 (Add both sides by -7) In bx = by, b is 5 in this problem. To get the
7–x–7=3–7 equation x = y just cover the number 5
Step 2
– x = – 4 (Divide both sides by -1) and the exponents on each side will be the
Thus x = 4 is the solution of the equation x = y.
equation 57 – x = 125 57 – x = 53 and that‟s how you get 7 – x = 3.
Solve the value of x.
57 – x = 125
Check the solution by substituting x to the
Step 3 33 + 1 = 81
equation.
34 = 81

Solve the following exponential inequalities. (2 points each)


16-17. 43x + 2 < 64
Change 64 to 43 and apply power of a
power to simplify.
43x + 2 < 64
Use property a where b > 1 and bx < by,
Step 1 43x + 2 < 43
then x < y.
3x + 2 < 3 (Using property a)
bx is 43x + 2 while by is 43 which will make x
< y same as 3x + 2 < 3.
3x + 2 < 3 (Add –2 on both sides)
3x + 2 – 2 < 3 – 2
3x < 1 (Divide both sides by 3)

(When you divide by a negative number, the sign


Step 2 of the inequality must flip) Solve for the value of x in the inequality.

Therefore the solution for 43x + 2 < 64 is


.
18-19. 10 > 100-2x – 5
x

Change 100 to 102 and apply power of a


x -2x – 5
10 > 100 power to simplify.
10x > (102)-2x – 5 Use property b where b > 1 and bx > by,
Step 1
10x > 10-4x – 10 then x > y.
x > – 4x – 10 (Using property b) bx is 10x while by is 10-4x – 10 which will
make x > y same as x > – 4x – 10.
x > – 4x – 10 (Add 4x on both sides)
x + 4x > – 4x – 10 + 4x
5x > –10 (Divide both sides by 5)
Solve for the value of x in the inequality.
Step 2
x>–2
Therefore the solution for 10x > 100-2x – 5
is x > – 2
20-21. 9x ≤ 92x – 1
Use property a where b > 1 and bx ≤ by,
9x ≤ 92x – 1 then x ≤ y.
Step 1
x ≤ 2x – 1 (Using property a) bx is 9x while by is 92x – 1 which will make
x ≤ y same as x ≤ 2x – 1.
x ≤ 2x – 1 (Add –2x on both sides)
x – 2x ≤ 2x – 1 – 2x
–x ≤ –1 (Divide both sides by -1)
Step 2 (When you divide by a negative number, the sign Solve for the value of x in the inequality.
of the inequality must flip)
x≥1
Therefore the solution for 9x ≤ 92x – 1 is x ≥ 1

Page 68
SELF-INSTRUCTIONAL PACKETS
GENERAL MATHEMATICS GRADE 11

School Teaching Dates/ Week


Sept. 28 – Oct. 2, 2020
PAMPANGA HIGH SCHOOL (Week 6)
Teacher GENERAL MATHEMATICS TEACHERS Quarter First

I. OBJECTIVES

A. Content Standards
The learner demonstrates understanding of key concepts of exponential functions.

B. Performance Standards
The learner is able to apply the concepts of exponential functions to formulate and
solve real-life problems with precision and accuracy.

C. Learning Competencies
The learner

1. determines the intercepts, zeroes, and asymptotes of an exponential function;


(M11GM-Ig-1) and
2. solves problems involving exponential functions, equations, and inequalities.
(M11GM-Ig-2)

D. Objectives
At the end of the lesson, the learners should be able to:

1. find the intercepts, zeroes, and asymptotes of an exponential function;


2. represent real-life situations using exponential functions; and
3. show appreciation to the topic by relating and solving real –life situations.

II. CONTENT
GRAPHING EXPONENTIAL FUNCTIONS

Learning Resources

A. Reference
General Mathematics Learner‟s Material pages 88-98
General Mathematics by Orlando A. Oronce pages 108-138

III. PROCEDURES WEEK 6: Day 1

A. Reviewing previous lesson or presenting the new lesson

Bonjour! Did you find the previous lesson easy? Before we continue our exploration of
exponential functions, try to answer the challenge below.

DIRECTIONS: Tell whether each statement is TRUE or FALSE. Put a check mark (✓) on the
appropriate box. Copy ALL and write the answers on your Gen Math notebook.

STATEMENTS TRUE FALSE


1. There is an integer x that will make x2 equal to 2x.
2. Every linear function has x-intercept.
3. A quadratic function has vertical asymptote.
4. A graph of y = x intersects the graph of y = –x at (1, 1)
5. 32 = –9

Page 69
Score
Description
Now, check your work by turning to page 78 for the key 5
Naol perfect!
to correction. How many correct answers did you get? Rate
4Bongga ka!
your result using the table on the right. If your score is at least 3
out of 5, you may now proceed to next part of the discussion. 2-3
Pwede na besh
Practice pa more
0-1
In this module, you will learn more about exponential on page 76 sec J
functions and their graphs. You will also solve problems involving exponential functions. Let‟s
get ready!

B. Establishing a purpose for the lesson

Reflect on this illustration:

Did you know that


exponential functions can predict the
future? Or why this model is massively
used in predicting the no. of possible
COVID cases for next 15 days.
Statisticians used this model so that
the stakeholders can determine
whether the restrictions for community
quarantine will be lessened or
tightened.

Our life is indeed so predictable and sometimes there are events in our life that we do
not expect to happen. Making an exponential model to it may help us decide our future with an
element of a surprise factor. Enjoy every second of our life, exponentially!
Image retrieved from:
https://me.me/i/i-did-not-expect-this-where-you-actually-end-up-0090b22875cf4bb3b405a5c3c68a77b5

C. Presenting examples/instances of the new lesson

Lesson 18: Graphing Exponential Functions


What does the graph of exponential function look like? Let us sketch some examples
and observe the characteristics of this function.

Problem 1. Sketch the graph of f(x) = 2x


Step 1. Construct a table of values of ordered pairs for the given function.

X -4 -3 -2 -1 0 1 2 3
f(x) 1/16 1/8 ¼ ½ 1 2 4 8

Step 2. Plot the points in the table and connect them using a smooth curve.

Page 70
It can be observed that the function is defined for all values of x, is strictly increasing,
and attains only positive y-values. As x decreases without bound, the function approaches 0,
i.e., the line y = 0 is a horizontal asymptote.

Problem 2. Sketch the graph of f(x) = (½)x

Step 1. Construct a table of values of ordered pairs for the given function.

X -3 -2 -1 0 1 2 3 4
f(x) 8 4 2 1 ½ ¼ 1/8 1/16

Step 2. Plot the points in the table and connect them using a smooth curve.

It can be observed that the function is defined for all values of x, is strictly decreasing,
and attains only positive values. As x increases without bound, the function approaches 0, i.e.,
the line y = 0 is a horizontal asymptote.

From the two examples, we can state the following properties.

PROPERTIES OF EXPONENTIAL FUNCTIONS:


1. The domain is the set of all real numbers.
2. The range is the set of all positive real numbers.
3. It is a one-to-one function. It satisfies the Horizontal Line Test.
4. The y-intercept is 1. There is no x-intercept.
5. The horizontal asymptote is the line y = 0 (or the x-axis). There is no vertical asymptote.
6. The function is increasing if b > 1, and is decreasing if 0 < b < 1.

Now, it‟s your turn to sketch the graph of an exponential function.

Score Description
5 Naol perfect!
DIRECTIONS: Sketch the graph of f(x) = . / . Sketch the graph 4 Bongga ka!
and write the solutions on your Gen Math notebook. (5 points) 2-3 Pwede na besh
Practice pa more
0-1
on page 76 sec J

Page 71
Now, check your work by turning to page 78 for the key to correction. How many correct
answers did you get? Rate your result using the table above. If your score is at least 3 out of 5,
you may now proceed to next part of the discussion.

WEEK 6: Day 2

D. Discussing new concepts and practicing new skills #1

Lesson 19: Solving Problems involving Exponential Functions

Before we start the discussion, try to answer this challenge:

DIRECTIONS: Tell whether each statement is TRUE or FALSE. Put a check mark (✓) on the
appropriate box. Copy ALL and write the answers on your Gen Math notebook.

STATEMENTS TRUE FALSE


1. In decimal form, is equal to 1.6.
2. In percent form, 12.0 is equal to 120%.
3. If the original price of ₱50.00 is increased by 20%, its new
price is ₱60.00
4. ( )
5.

Now, check your work by turning to page 79 for the key Score Description
to correction. How many correct answers did you get? Rate 5 Naol perfect!
your result using the table on the right. If your score is at least 3 4 Bongga ka!
out of 5, you may now proceed to next part of the discussion. 2-3 Pwede na besh
Practice pa more
The graph of an exponential function defined by 0-1
on page 77 sec J
( ) where and indicates that:

 It is an increasing function, sometimes called an exponential growth function if .


 It is a decreasing function, sometimes called an exponential decay function if .

Try to explore this problem:

When a quantity increases by 20%, how does its new value compare to its original
value? That is, what is the ratio of the new value to the original value? See the table below to
discover the pattern:

Original Value New Value (₱) Ratio of the New Value to Original Value
(₱) Increased by 20% Fraction Decimal Percent

Page 72
In the given problem, for a 20% increase to any original value, the ratio of the new value
to the original value is always the same 120%. This ratio is called the growth
factor associated with the specified percent increase.

The growth factor is formed by adding the specified percent increase to 100% and then
changing this percent to its decimal form. The growth factor in the given problem that
corresponds to a 20% increase is . But, in decimal form.

Score Description
DIRECTIONS: Determine the growth factor of the quantity that 8 Naol perfect!
increases by the given percent. Copy ALL and write the 6-7 Bongga ka!
answers on your Gen Math notebook. (2 points each)
4-5 Pwede na besh
1–2. 50% 5–6. 10% Practice pa more
0-4
3–4. 75% 7–8. 12.5% on page 77 sec J

Now, check your work by turning to page 79 for the key to correction. How many correct
answers did you get? Rate your result using the table above. If your score is at least 4 out of 8,
you may now proceed to next part of the discussion.

WEEK 6: Day 3

E. Discussing new concepts and practicing new skills #2

Now, let us consider solving word problems involving the growth factor of an exponential
function.

Problem 1: Emerson deposits ₱50, 000.00 in a savings account. The account pays 6% annual
interest. If he makes no more deposits and no withdrawals, calculate his new balance after 10
years.

Solution:
The interest represents a 6% rate of growth per year, so the growth factor or the
constant multiplied is:
100% + 6% = 106% or 1.06

To find an equation that can be used to find the new balance after any number of years
by considering the yearly calculations, we have:

Expanded Form Exponential Form Result


Initial Amount: ₱50, 000 ₱50, 000
After 1 year: ₱50, 000(1.06) ₱50, 000(1.06)1 ₱53, 000
2 years: ₱50, 000(1.06)(1.06) ₱50, 000(1.06)2 ₱56, 180
3 years: ₱50, 000(1.06)(1.06)(1.06) ₱50, 000(1.06)3 ₱59, 551
4 years: ₱50, 000(1.06)(1.06)(1.06)(1.06) ₱50, 000(1.06)4 ₱63, 124
x years: ₱50, 000(1.06)(1.06)(1.06) … (1.06) ₱50, 000(1.06)x

We can now use this equation ( ) where x represents time in years


and y represents the new balance in pesos. To find the new balance after 10 years, we have:

( )

The new balance after 10 years is ₱89, 542.38.

When an amount increases by a certain percentage, we have exponential growth.

Page 73
REMEMBER ME!

The rule for exponential growth can be modeled by:


y = abx
where a is the starting number, b is the growth factor, and x is the number of intervals
(minutes, years, and so on).

Problem 2: The novel corona virus spreads in the urbanized community at a rate of 25% each
day. There are 500 confirmed COVID-19 cases as of today. How many possible COVID-19
cases will there be

a. tomorrow? b. one week from now? c. one month from now?

Solution:

a. Using the formula y = abx where a = 500, b = 1.25, and x = 1, then

y = 500(1.25)1
y = 625

There will be 625 COVID-19 cases tomorrow.

b. Using the formula y = abx where a = 500, b = 1.25, and x = 7, then

y = 500(1.25)7
y = 2, 384.19

There will be 2, 385 COVID-19 cases one week from now.


(NOTE: Round-up to the next whole number)

c. Using the formula y = abx where a = 500, b = 1.25, and x = 30, then

y = 500(1.25)30
y = 403, 896.78

There will be 403, 897 COVID-19 cases one month from now.
(NOTE: Round-up to the next whole number)

Score Description
DIRECTIONS: Solve the given exponential problem. Copy ALL 5 Naol perfect!
and write the answers on your Gen Math notebook. 4 Bongga ka!
(5 points)
2-3 Pwede na besh
Mrs. Lacap bought an antique dresser for ₱15, 000.00. She Practice pa more
0-1
estimates that it will increase in value by 5% per year. on page 77 sec J

1-2. Formulate an equation to calculate the value, y, of Mrs. Lacap‟s dresser after x years.
3. Find the value of the dresser after 3 years.
4. Find the value of the dresser after 5 years.
5. Find the value of the dresser after 8 years.

Now, check your work by turning to page 79 for the key to correction. How many correct
answers did you get? Rate your result using the table above. If your score is at least 3 out of 5,
you may now proceed to next part of the discussion.

Page 74
WEEK 6: Day 4

F. Developing mastery

GENERAL DIRECTIONS: Copy ALL and write the answers on your Gen Math Notebook.

GRAPHS OF EXPONENTIAL FUNCTIONS

DIRECTIONS: In only one Cartesian plane, sketch the graphs of:

1-2. y = 3x
3-4. y = (3)-x
5-6. y = (-3)x
7-9. Compare the graph of y = 3x with the two other graphs.

CREATE YOUR OWN EXPONENTIAL PROBLEM


DIRECTIONS: Form an exponential problem that could be modeled by the equation
y = 5000(1.04)x.

10 – 12. Own Problem


13. Answer the problem when x = 4
14. Answer the problem when x = 7
15. Answer the problem when x = 10

Now, check your work by turning to page 80 for the key to correction. How many correct
answers did you get?

G. Finding practical applications of concepts and skills in daily living

The Natural Exponential Function

While an exponential function may have various


bases, a frequently used base is the irrational
number e, whose value is approximately 2.71828.
Because e is a commonly used base, the natural exponential function
is defined having e as the base.

Euler's number has many practical uses, particularly in higher


level mathematics such as calculus, differential equations, discrete
mathematics, trigonometry, complex analysis, statistics, among others.

The natural exponential function is the function f(x) = ex.

H. Making generalizations and abstractions about the lesson

The graph of an exponential function defined by ( ) where and indicates


that:

 It is an increasing function, sometimes called an exponential growth function if .


 It is a decreasing function, sometimes called an exponential decay function if .

The rule for exponential growth can be modeled by: y = abx.

Page 75
WEEK 6: Day 5

I. Evaluating learning

GENERAL DIRECTIONS: This part is recorded and graded. Copy ALL the given using this
format on a separate graphing paper. The graphing paper will be submitted to your Math
teacher on the day of the scheduled retrieval.

ACTIVITY 6.1 IN GENERAL MATHEMATICS


(WEEK 6)
Name: _________________Section: _________ Parent‟s Signature: ___________________

GRAPHING EXPONENTIAL FUNCTIONS


DIRECTIONS: Sketch the graph of the following exponential functions by following the two
steps. (5 points each)

1. ( )
2. ( )
3. ( )
4. ( )
5. ( )

ACTIVITY 6.2 IN GENERAL MATHEMATICS


(WEEK 6)
Name: _________________Section: _________ Parent‟s Signature: ___________________

SOLVING EXPONENTIAL PROBLEMS


DIRECTIONS: Solve the following exponential problems. Continue writing your solutions in the
graphing paper. (5 points each)

1. The price of fuel has been rising with inflation at 4% per year. If a liter of diesel was ₱43.00 a
year ago, what is it now?

2. The value of your car depreciates by 8% per year. If the car is worth ₱780, 000 today, what
will it worth in 5 years?

3. Mexico has a population of around 100 million people, and it is estimated that the population
will double in 21 years. If population growth continues at the same rate, what will be the
population 15 years from now?

J. Additional activities for application or remediation


GENERAL DIRECTIONS: Copy ALL and write the answers on your Gen Math notebook.

Remedial Activity 6.1: TRUE or FALSE

DIRECTIONS: Tell whether each statement is TRUE or FALSE. Put a check mark (✓) on the
appropriate box.

STATEMENTS TRUE FALSE


1. There is an integer x that will make x3 equal to 3x.
2. Every linear function has y-intercept.
3. A quadratic function has horizontal asymptote.
4. A graph of y = x intersects the graph of y = –x at (0, 0)
5.

Remedial Activity 6.2: GRAPHING EXPONENTIAL FUNCTIONS

DIRECTIONS: Sketch the graph of f(x)=( ) . Sketch the graph and write the solutions on your
Gen Math notebook. (5 points)

Page 76
Remedial Activity 6.3: EXPONENTS, DECIMALS, PERCENTAGE

DIRECTIONS: Tell whether each statement is TRUE or FALSE. Put a check mark (✓) on the
appropriate box. Copy ALL and write the answers on your Gen Math notebook.

STATEMENTS TRUE FALSE


1. In decimal form, is equal to 3.1.
2. In percent form, 2.0 is equal to 200%.
3. If the original price of ₱60.00 is increased by 30%, its new
price is ₱87.00
4. ( )
5.

Remedial Activity 6.4: GROWTH FACTOR

DIRECTIONS: Determine the growth factor of the quantity that increases by the given percent.
Copy ALL and write the answers on your Gen Math notebook. (2 points each)

1–2. 25% 5–6. 15%


3–4. 65% 7–8. 7.5%

Remedial Activity 6.5: PROBLEM-SOLVING

DIRECTIONS: Solve the given exponential problem. (5 points)


When you use credit card to purchase an item, you are actually making a loan. A
constant percent interest is added to the balance. Maricar buys a microwave worth ₱7, 500.00
with her credit card. The balance grows 3% each month. How much will she owe if she makes
no payments in 6 months.

Page 77
APPENDIX A
KEY TO CORRECTION

CHALLENGE 6.1:

STATEMENTS TRUE FALSE


1. There is an integer x that will make x2 equal to 2x. ✓
2. Every linear function has x-intercept. ✓
3. A quadratic function has vertical asymptote. ✓
4. A graph of y = x intersects the graph of y = –x at (1, 1) ✓
5. 32 = –9 ✓

Explanation:

1. When x = 2, x2 and 2x are equal. That is,


(2)2 = 2(2)
4=4
2. The graph of a linear function will have both x and y-intercepts. Therefore, a slant line always
touches the x-axis.
3. Quadratic functions do not have asymptotic behavior. Therefore, it is impossible to have a
vertical asymptote.
4. The graphs of y = x and y = –x intersects at the origin (0, 0) not at point (1, 1).
5. To correct the expression, it should 3 2 = 9. The square of any number is always positive.

CHALLENGE 6.2:
Sketch the graph of f(x) = . / .
Step 1. Construct a table of values of ordered pairs for the given function.

X -1 0 1 2
f(x) 4 1 ¼ 1/16

Step 2. Plot the points in the table and connect them using a smooth curve.

Page 78
CHALLENGE 6.3:

STATEMENTS TRUE FALSE


1. In decimal form, is equal to 1.6. ✓
2. In percent form, 12.0 is equal to 120%. ✓
3. If the original price of ₱50.00 is increased by 20%, its new

price is ₱60.00
4. ( ) ✓
5. ✓

CHALLENGE 6.4:

1–2. For 50%: 100% + 50% = 150% = 1.50


3–4. For 75%: 100% + 75% = 175% = 1.75
5–6. For 10%: 100% + 10% = 110% = 1.10
7–8. For 50%: 100% + 12.5% = 112.5% = 1.125

CHALLENGE 6.5:

Mrs. Lacap bought an antique dresser for ₱15, 000.00. She estimates that it will increase in
value by 5% per year.

1-2. Equation: y = 15 000 (1.05)x

3. Using the formula y = abx where a = 15 000, b = 1.05, and x = 3, then

y = 15, 000 (1.05)3


y = 17, 364.375

The dresser‟s cost will be ₱ 17, 365.

4. Using the formula y = abx where a = 15 000, b = 1.05, and x = 5, then

y = 15, 000 (1.05)5


y = 19, 144.22

The dresser‟s cost will be ₱ 19, 145.

5. Using the formula y = abx where a = 15 000, b = 1.05, and x = 8, then

y = 15, 000 (1.05)8


y = 22, 161.83

The dresser‟s cost will be ₱ 22, 162.

Page 79
APPENDIX B
KEY TO CORRECTION
F. Developing mastery

GRAPHS OF EXPONENTIAL FUNCTIONS


DIRECTIONS: In only one Cartesian plane, sketch the graphs of:

1-2. y = 3x 3-4. y = (3)-x 5-6. y = (-3)x

y = (3)-x y = 3x

y = (-3)x

7-9. Compare the graph of y = 3x with the two other graphs.

The two other graphs are mirror images of y = 3 x.


The graph of y = (3)-x is a reflection of y = 3x about the y-axis.
The graph of y = (-3)x is a reflection of y = 3x about the x-axis.

CREATE YOUR OWN EXPONENTIAL PROBLEM


DIRECTIONS: Form an exponential problem that could be modeled by the equation
y = 5000(1.04)x.

10 – 12. (Answers may vary)

13. Answer the problem when x = 4


y = 5000(1.04)4
y = 5849.29
y = 5850
14. Answer the problem when x = 7
y = 5000(1.04)7
y = 6579.66
y = 6580
15. Answer the problem when x = 10
y = 5000(1.04)10
y = 7401.22
y = 7402

Page 80
SELF-INSTRUCTIONAL PACKETS
GENERAL MATHEMATICS GRADE 11

School Teaching Dates/ Week


October 6 – 13, 2020
PAMPANGA HIGH SCHOOL (Week 7)
October 5, 2020 – World Teacher’s Day
Teacher GENERAL MATHEMATICS TEACHERS Quarter First

I. OBJECTIVES

A. Content Standards
The learner demonstrates understanding of key logarithmic functions.

B. Performance Standards
The learner is able to apply the concepts of logarithmic functions to formulate and
solve real-life problems with precision and accuracy.

C. Learning Competencies
The learner
1. represents real-life situations using logarithmic functions; (M11GM-Ih-1)
2. distinguishes logarithmic function, logarithmic equation, and logarithmic
inequality; (M11GM-Ih-2)
3. illustrates the laws of logarithms; (M11GM-Ih-3)
4. solves logarithmic equations and inequalities; (M11GM-Ih-i-1)
5. represents a logarithmic function through its: (a) table of values, (b) graph, and
(c) equation; (M11GM-Ii-2)
6. determines the intercepts, zeroes, and asymptotes of logarithmic functions;
(M11GM-Ii-4) and
7. solves problems involving logarithmic functions, equations, and inequalities.
(M11GM-Ij-2)

D. Objectives
At the end of the lesson, you should be able to:
1. define logarithmic function, logarithmic equation, and logarithmic inequality;
2. convert expressions in logarithmic form to exponential form and vice versa;
3. apply the laws of logarithms in simplifying and expanding logarithms;
4. solve logarithmic equations and inequalities;
5. represent logarithmic functions through tables, graphs, and equations;
6. illustrate the characteristics of the graph of a logarithmic function; and
7. solve problems involving logarithmic functions.

II. CONTENT
LOGARITHMIC FUNCTIONS

Learning Resources

A. Reference
General Mathematics Learner‟s Material pp. 99-124
General Mathematics (Orlando Oronce) pp. 129-171

III. PROCEDURES WEEK 7: Day 1

A. Reviewing previous lesson or presenting the new lesson

The last function that you need to learn is in this chapter. Before anything else, you need
to answer the challenge below to see if you remember the important points about exponential
function.

Page 81
DIRECTIONS: Match column A to column B. Write the letter on the blank provided.
Copy ALL and write the answers on your Gen Math notebook. (1 point each)
Column A Column B
_____ 1. y = 5x A. exponential growth
_____ 2. 4x+1 = 2x-3 B. half life
_____ 3. Given 2x = 64, find x. C. doubling time
_____ 4. The number of covid patients increases by E. exponential equation
5% each week. F. exponential function
_____ 5. It is the time it takes for half of the G. 6
substance to decay. H. 5

Now, check your work by turning to page 91 for the key


to correction. How many correct answers did you get? If your Score Description
score is at least 2 out of 5, you may now proceed to next part of 5 Naol perfect!
the discussion. 4 Bongga ka!
WEEK 7: Day 2 2-3 Pwede na besh
Practice pa more
B. Establishing a purpose for the lesson 0-1
on page 90 sec J

MATH TRIVIA

The word 'logarithm' was coined by John


Napier, the inventor of a form of logarithm, in 1614. It
comes from the Greek words 'logos' which means
ratio and 'arithmos' which means number.

C. Presenting examples/instances of the new lesson

Lesson 20: Definition of Logarithm


A logarithm is an exponent which base b must be raised to produce x. See the
conversion below.

The conversion says that the equation by = x in exponential form can be converted into an
equation in logarithmic form log b (x) = y and vice versa. This is true for b > 0, b ≠ 1 and x > 0.
Things to remember:
 In both logarithmic and exponential forms, b is the base.
 In the exponential form, y is an exponent. But y = log b (x). This implies that the
logarithm is actually an exponent.
 In the logarithmic form log b (x), x cannot be negative. For example log 10 (-100) is
undefined since 10 raised to any exponent will never result to a negative number.
 The value of logb (x) can be negative. For example, log 10 (0.1) = -1 because
10-1 = 0.1.

Page 82
Illustrative Example 1: Exponential to Logarithmic Conversion

Exponential Form: 102 = 100


Logarithmic Form: log10 (100) = 2

Note: You can check if your conversion is correct using a scientific calculator.
Just input log10 (100) or simply log (100) and press the equal sign. The
answer should be 2.

Illustrative Example 2: Logarithmic to Exponential Conversion

Logarithmic Form: log2 (16) = 4


Exponential Form: 24 = 16

DIRECTIONS: Complete the table below by converting the given equation in logarithmic form to
exponential form and vice versa. Copy ALL and write the answers on your Gen Math notebook.
(1 point each)

LOGARITHMIC FORM EXPONENTIAL FORM

log4 (64) = 3 1. _____________

2. _____________ 34 = 81

log10 (0.01) = -2 3. _____________

4. _____________ ey = 9

logD (K) = 7 5. _____________

Now, check your work by turning to page 91 for the key Score Description
to correction. How many correct answers did you get? If your 5 Naol perfect!
score is at least 2 out of 5, you may now proceed to next part of 4 Bongga ka!
the discussion. 2-3 Pwede na besh
Practice pa more
0-1
on page 90 sec J
Additional Information:

1. Common logarithm – is the logarithm with base 10 and is usually written in log or log 10.

Examples: log10 100


log 20
log (x - 5)

2. Natural Logarithm – is the logarithm with base e and is usually written in log e or ln
(pronounced as „el en‟). Euler‟s number, e, is a constant with approximate value of 2.71828.
You can find e in your scientific calculator.

Examples: loge 27
ln 9x
ln 44

3. Change-of-base Property – can be used when evaluating logarithms. Some old model
calculators only have the LOG button function (common logarithm). If the logarithm given has a
base other than 10, you still need to change its base in order to evaluate it.

Page 83
Example: Press log2 (50) = in your calculator
New calculators can input the given logarithm as is and the answer is 5.64
But for old calculators, the base should be changed first like this,
( )
log2 (50) will become . Base 2 was changed into base 10 (common
( )
logarithm) and the expression was expressed as a ratio.
( )
By pressing = in the calculator, the answer is also 5.64.
( )
Thus, using the change-of-base property, log2 (50) = .
WEEK 7: Day 3

D. Discussing new concepts and practicing new skills #1

Lesson 21: Logarithmic Function: y = logb (x)

Did you know that logarithmic function is related to exponential function? Try to see their
relationship by getting the inverse of the exponential function y = b x.
Given y = bx
Interchange the variables x = by
Solve for y using the exp-log conversion logb (x) = y or
y = logb (x)
Thus, the logarithmic function y = log b (x) is the inverse of the exponential function y = b x
and vice versa. See the graph below. As previously discussed in week 4, when you graph
functions that are inverses of each other, the two curves will form a mirror image with an axis of
symmetry at the line y = x.
Exponential Function y = 2x
x -2 0 1 2
y = 2x ¼ 1 2 4
x
Logarithmic Function y = log (x) y=2 y = log (x)
2 2

X ¼ 1 2 4
y = log2 x -2 0 1 2
y=x

Characteristics of the Graph of a Logarithmic


Function y = logb (x)

1. The x-intercept is 1. There is no y-intercept.


2. The y-axis is a vertical asymptote.
3. Domain is all positive real numbers. Range is all real numbers.
4. If b>1, the function is increasing. If 0<b<1, the function is decreasing.

y = log (x)
2 y = log (x)
1/2

Increasing Logarithmic Function Decreasing Logarithmic Function

Page 84
Lesson 22: Logarithmic Function, Logarithmic Equation, and Logarithmic Inequality

Logarithmic
Logarithmic Function Logarithmic Equation
Inequality
An equation in the form An equation involving An inequality involving
Definition
f(x) = logb (x) logarithms logarithms
f(x) = log3 (2x) log2 (x) = - 4 log3 (x) ≤ 5
Examples
y = log (x2) log4 (2x) = log4 (10) log4 (3x+2) < log4 (x+5)

DIRECTIONS: Determine if the given mathematical statement is a logarithmic function,


logarithmic equation, or logarithmic inequality. Copy ALL and write the answers on your Gen
Math notebook. (1 point each)

________________A) g(x) = log6 5x Score Description


________________B) log5 (x+8) = 3 5 Naol perfect!
________________C) log2 x + log2 (x-6) = 4 4 Bongga ka!
________________D) log5 x > 4 2-3 Pwede na besh
________________E) y = log (x-2) Practice pa more
0-1
on page 90 sec J

Now, check your work by turning to page 91 for the key to correction. How many correct
answers did you get? If your score is at least 2 out of 5, you may now proceed to next part of
the discussion.

E. Discussing new concepts and practicing new skills #2

Lesson 23: Solving Logarithmic Equations

A logarithmic equation is an equation containing a variable in a logarithmic expression.


Examples: log5 (x+4) = 6 log5 (x+8) ln 3x = 2

Tips in solving logarithmic equations:


1. Isolate the logarithmic term on one side of the equation then rewrite it to exponential
form.
2. Apply the one-to-one property of logarithmic functions: if log b (x) = logb (y), then x = y.
3. Check for possible extraneous solutions (values of x that make the equation undefined).

Illustrative Examples: Solve the following logarithmic equations.

(1) log3 (x) = 5

Solution: 35 = x
x = 243
NOTE FOR (1) and (2): Convert the equation into its
(2) log5 (x+8) = 3 exponential form and then solve for x.

Solution: 53 = x + 8 NOTE FOR (3): Since there are logarithmic expressions with
125 – 8 = x the same base on both sides of the equation, one-to-one
x = 117 property should be used. Rewrite the equation by removing
the log symbols while leaving the expression in the
(3) log4 (2x) = log4 (10) parenthesis for both sides. Lastly, solve for the value of x.
Solution: 2x = 10
2x/2 = 10/2
x=5

Page 85
WEEK 7: Day 4

Lesson 24: Solving Logarithmic Inequalities


A logarithmic inequality is an inequality that involves logarithms.
Examples: log3 x ≤ 5 log4 (3x+2) < log4 (x+5)

Properties of Logarithmic Inequalities


For base greater than 1 (b > 1), For base between 0 and 1 (0 < b < 1),
if logb (x) > logb (y), then x > y if logb x > logb y, then x < y
if logb (x) > y, then x > by if logb x > y, then x < by
Unlike solving logarithmic equations, there are steps that you need to follow to solve
logarithmic inequalities.
STEP 1: Ensure that the logarithms are defined. Set the expression of the logarithm greater
than 0 and solve for x.
STEP 2: Solve for x in the given inequality using the properties.
STEP 3: Get the intersection of the inequalities obtained in steps A and B because this will be
the set of solutions.

Illustrative Examples: Solve the following logarithmic inequalities.

(1) log3 (x) ≤ 5

Solution:
Step 1: x > 0 NOTE for STEP 2: Using the property for base greater than 1 (if
Step 2: x ≤ 35 logb (x) > y, then x > by), express the logarithmic inequality into
x ≤ 243 exponential inequality and solve for x.
Step 3: 0 < x ≤ 243

(2) log0.2 (x) > 3


NOTE for STEP 2: Using the property for base less than 1 (if logb
Solution: x > y, then x < by), express the logarithmic inequality into
Step 1: x > 0
exponential inequality and solve for x. Remember that the
Step 2: x < 0.23
x < 0.008 inequality symbol must be flipped when applying this property.
Step 3: 0 < x < 0.008

(3) log6 (x+8) ≤ 2

Solution:
Step 1: x+8>0
x > -8 NOTE for STEP 2: Using the property for base greater than 1, log6
Step 2: x+8 ≤ 62 (x+8) ≤ 2 will become x+8 ≤ 62. After solving x, the inequality is x ≤ 28.
x ≤ 28
Step 3: -8 < x ≤ 28

Lesson 25: Applications of Logarithmic Functions (Logarithmic Models)

1. Determining the pH level of a


Solution:

The pH (potential of hydrogen) level


of a water-based solution is defined
as pH = - log [H+]
where [H+] is the concentration of
hydrogen ions in moles per liter.
Solutions are acidic if their pH is less
than 7 and they are basic if their pH
is more than 7.

Page 86
Illustrative Example 1: A 1-liter solution contains 0.00001 moles of hydrogen ions. Find its pH
level using your scientific calculator.

Solution: pH = -log[H+]
pH = -log(0.00001) or - log (10-5)
pH = -(-5)
pH = 5
Thus, the pH level of the solution is 5.

2. Determining the Sound Level

In acoustics, the decibel (dB) level of a sound is


( )
where:
I - sound intensity in watts/m2
10-12 - least audible sound a human can hear

Illustrative Example 2: A certain sound wave has an intensity of 10 -5 w/m2, find its
corresponding decibel value. Use your scientific calculator.

Solution: . /

( )

Thus, the sound level of a certain sound is 70 dB.

3. Determining Earthquake Magnitude (Richter Scale)

The magnitude M of an
earthquake is given by

( )

where:
E (in joules) – energy released by the
earthquake
104.4 – energy released by a very small
reference earthquake

The chart on the right describes the


magnitude of the earthquake per level.
Each magnitude level is ten times
stronger than the previous level!

Illustrative Example 3: Suppose that an earthquake released approximately 10 12 joules of


energy. What is its magnitude on a Richter scale? Use your scientific calculator.

Solution:

( ) ( )

Thus, the earthquake has a magnitude of 5.07.

Page 87
F. Developing Mastery WEEK 7: Day 5

Now, it‟s your time to solve logarithmic equations and inequalities! You must finish the
challenges below to master this lesson. Good luck!

Score Description
DIRECTIONS: Solve the following logarithmic equations. Copy 10 Naol perfect!
ALL and write the answers on your Gen Math notebook.
(5 points each) 7-9 Bongga ka!
1. log4 (x + 1) = 2 4-6 Pwede na besh
2. log3 (x+4) = log3 (2x-4) Practice pa more
0-3
on page 90 sec J

Now, check your work by turning to page 91 for the key to correction. How many correct
answers did you get?

Score Description
DIRECTIONS: Solve the following logarithmic inequalities. Copy 10 Naol perfect!
ALL and write the answers on your Gen Math notebook. 7-9 Bongga ka!
(5 points each)
4-6 Pwede na besh
1. log0.5 (x) > 1 Practice pa more
0-3
2. log2 (x+18) ≥ 4 on page 90 sec J

Now, check your work by turning to page 91 for the key to correction. How many correct
answers did you get?

DIRECTIONS: Answer the following problems involving logarithmic functions on your math
notebook. Copy ALL and write the answers on your Gen Math notebook.(5 points each)

1. A fruit juice has a concentration of 0.00158 moles of Score Description


hydrogen ions. Find its pH level. 15 Naol perfect!
2. An earthquake in Zambales released 7.50x10 14 joules of 12-14 Bongga ka!
energy. What is the magnitude of the earthquake?
3. A loud explosion in the city has a sound intensity of 7 w/m2, 8-11 Pwede na besh
find its corresponding sound level in dB. Practice pa more
0-7
on page 90 sec J

Now, check your work by turning to page 91 for the key to correction. How many correct
answers did you get? If your score is at least 8 out of 15, you may now proceed to next part of
the discussion.

G. Finding practical applications of concepts and skills in daily living

Did you know that logarithms have applications in real life? Here are some
examples:

 Logarithms are used to express large quantities or values.


 Logarithms can be used in solving exponential equations.
 Measuring sound intensity (decibel measures)
 Measuring earthquake intensity/magnitude (Richter scale)
 Measuring the brightness of stars
 Measuring the pH of a solution

Page 88
H. Making generalizations and abstractions about the lesson

To summarize the lesson, you can say that:


 Logarithmic function is a function of the form f(x) = log b (x).
 Logarithmic equation is an equation which includes logarithmic expressions and
has = sign.
 Logarithmic inequality is an inequality which includes logarithmic expressions
and has <, >, ≤, or ≥ sign.
 The exponential function f(x) = b x is the inverse of the logarithmic function
f(x) = logb (x) and vice versa.
 To solve logarithmic equations, isolate the logarithmic term on one side of the
equation then rewrite it to exponential form.
 To solve logarithmic inequalities, we can apply the properties of logarithmic
inequalities for b > 1 and 0 < b < 1.
 Logarithmic functions are used in measuring earthquake magnitude, sound level,
and pH of a solution.

I. Evaluating Learning WEEK 7: Day 6

GENERAL DIRECTIONS: This part is recorded and graded. Copy ALL the given using this
format on a separate yellow paper. The yellow paper will be submitted to your Math teachers
on the day of the scheduled retrieval.

ACTIVITY 7.1 IN GENERAL MATHEMATICS


(WEEK 7)

Name: _________________Section: _________ Parent‟s Signature: ___________________

LOGARITHMIC-EXPONENTIAL CONVERSION
DIRECTIONS: Convert the following expressions to logarithmic or exponential form.
(5 points each)
1. log2 (64) = x Exponential Form: ____________
2. logx (1) = 0 Exponential Form: ____________
3. 1 = Logarithmic Form: ____________
4. Logarithmic Form: ____________

ACTIVITY 7.2 IN GENERAL MATHEMATICS


(WEEK 7)

Name: _________________Section: _________ Parent‟s Signature: ___________________

LOGARITHMIC FUNCTIONS, EQUATIONS, AND INEQUALITIES


DIRECTIONS: Identify if the given statement is a logarithmic function, equation, or inequality.
(5 points each)
1. log5 (x) > 4 _________________
2. f(x) = ln (x-3) _________________
3. log6 (4x+8) = 2 _________________
4. log (6/x) > log (x+5) _________________

ACTIVITY 7.3 IN GENERAL MATHEMATICS


(WEEK 7)

Name: _________________Section: _________ Parent‟s Signature: ___________________

LOGARITHMIC EQUATIONS
DIRECTIONS: Solve the following equations. Show your complete solution and box your final
answer. (10 points each)
1. log4 (x+1) = 2
2. log3 (x+4) = log3 (2x-4)

Page 89
ACTIVITY 7.4 IN GENERAL MATHEMATICS
(WEEK 7)
Name: _________________Section: _________ Parent‟s Signature: ___________________

LOGARITHMIC INEQUALITIES
DIRECTIONS: Solve the following inequalities. Show your complete solution and box your final
answer. (10 points each)
1. log0.5 (x) > 1
2. log2 (x+18) ≥ 4

ACTIVITY 7.5 IN GENERAL MATHEMATICS


(WEEK 7)
Name: _________________Section: _________ Parent‟s Signature: ___________________

LOGARITHMIC MODELS
DIRECTIONS: Solve the following problems involving logarithmic functions. (5 points each)
1. Find the sound level of a whisper with a sound intensity of 5.2 x 10-10 watts per square meter.
2. The water of a hot spring in Japan has a concentration of 0.000000369 moles per liter. What
is the pH level of the water?
3. A strong earthquake in Samar released 8.30x10 15 joules of energy. What is the magnitude of
the earthquake? Round-off your answer to two decimal places.

J. Additional activities for application or remediation


GENERAL DIRECTIONS: Copy ALL and write the answers on your Gen Math notebook.

Remedial Activity 7.1: LOGARITHMIC-EXPONENTIAL CONVERSION


DIRECTIONS: Convert the following expressions to logarithmic or exponential form.
1. log4 (1024) = 5 Exponential Form: ____________
2. logR (A) = T Exponential Form: ____________
3. 83 = 512 Logarithmic Form: ____________
4. CE = V Logarithmic Form: ____________

Remedial Activity 7.2: LOGARITHMIC FUNCTIONS, EQUATIONS, AND INEQUALITIES


DIRECTIONS: Identify if the given statement is a logarithmic function, equation, or inequality.
1. log0.6 (x) =8 _________________
2. y = log (2x3) _________________
3. log5 (3x-4) = 10 _________________
4. log4 (x + 7) > 3 _________________

Remedial Activity 7.3: LOGARITHMIC EQUATIONS


DIRECTIONS: Solve the following equations. Show your complete solution and box your final
answer.
1. ln (2x) = 3
2. log (2x - 1) = log (x + 3)

Remedial Activity 7.4: LOGARITHMIC INEQUALITIES


DIRECTIONS: Solve the following inequalities. Show your complete solution and box your final
answer.
1. log5 (x) < 2
2. log4 (x + 7) > 3

Remedial Activity 7.5: LOGARITHMIC MODELS


DIRECTIONS: Solve the following problems involving logarithmic functions.
1. Find sound level of a rock concert with an intensity of 3.44 watts per square meter.
2. A solution has a concentration of 0.00000246 moles of hydrogen ions. Find its pH level.
3. The 1906 San Francisco earthquake released approximately of 5.96x10 16 energy. Find the
magnitude of the earthquake. Round your answer to two decimal places.

Page 90
APPENDIX A
Key to Correction

CHALLENGE 7.1: CHALLENGE 7.2:


1. F
2. E LOGARITHMIC FORM EXPONENTIAL FORM
3. G
4. A log4 (64) = 3 1. 43 = 64
5. B
2. log3 (81) = 4 34 = 81

log10 (0.01) = -2 3. 10-2 = 0.01


CHALLENGE 7.3:
4. loge (9) = y ey = 9
1. Function
2. Equation logD (K) = 7 5. D7 = K
3. Equation
4. Inequality
5. Function

CHALLENGE 7.4: (5 pts each: 5 pts – correct, 3 pts – wrong, 0 pt – no answer)


Solve the following logarithmic equations.

1. log4 (x + 1) = 2 2. log3 (x+4) = log3 (2x-4)


Solution: Solution:
42 = x + 1 x + 4 = 2x – 4
16 = x + 1 4 + 4 = 2x – x
16 – 1 = x 8=x
x = 15 x=8

CHALLENGE 7.5: (5 pts each: 5 pts – correct, 3 pts – wrong, 0 pt – no answer)


Solve the following logarithmic inequalities.

1. log0.5 (x) > 1 2. log2 (x+18) ≥ 4


Solution: Solution:
Step 1: x > 0 Step 1: x + 18 > 0
Step 2: x < 0.51 (flipped > to <) x > -18
x < 0.5 Step 2: x ≥ 16 - 18
Step 3: 0 < x < 0.5 x ≥ -2
Step 3: x ≥ -2

CHALLENGE 7.6: (5 pts each: 5pts – correct, 3 pts – wrong, 0 pt – no answer)


Answer the following problems involving logarithmic functions.

1. pH = -log[H+] 𝐼
pH = -log(0.00158) 3. 𝐷 . /
pH = 2.80
𝐸 . /
2. 𝑀 . /

𝟏𝟐𝟖 𝟒𝟓 𝒅𝑩
𝑀 ( )

𝑴 𝟔 𝟗𝟖

Page 91

You might also like